ORTHOPEDIC MCQS ONLINE QUESTION BANK H3D

ORTHOPEDIC MCQS ONLINE QUESTION BANK H3D

3930. (3857) Q9-7640:

Which of the following describes the histologic appearance of a schwannoma:

 

1) Herringbone pattern of fibroblasts and collagen

3) Storiform pattern of spindle cells and giant cells

2) Areas of cellular twisted nuclei and separate myxoid areas

5) Nests of cells with clear cytoplasm and areas of pigment deposition

4) Biphasic â Spindle cells and epithelial cells

 

Schwannomas have a characteristic histologic appearance:

 

 

Antoni A: Cellular areas with twisted nuclei (the cells are Schwann cells) Antoni B: Myxoid areas

 

 

Verocay bodies are also seen and are characteristic of schwannomas Palisading nuclei in a parallel arrangement

 

The other responses refer to: Fibrosarcoma: Herringbone pattern

 

Malignant fibrous histiocytoma: Storiform pattern

 

Synovial sarcoma: Biphasic â epithelial cells and spindle cells

 

 

Clear cell sarcoma: Nests of cells with clear cytoplasm and pigment deposition Correct Answer: Areas of cellular twisted nuclei and separate myxoid areas

 

3931. (3858) Q9-7641:

Which of the following immunochemical stains is typically positive in schwannomas:

 

1) Keratin

3) Myoglobin

2) Desmin

5) Epithelial membrane antigen

4) S-100

 

Schwannomas have a characteristic histologic appearance:

 

 

Antoni A: Cellular areas with twisted nuclei (the cells are Schwann cells) Antoni B: Myxoid areas

 

 

Verocay bodies are also seen and are characteristic of schwannomas Palisading nuclei in a parallel arrangement

The other responses refer to:

 

 

Keratin: Stains epithelial cells (positive in synovial sarcoma and epitheloid sarcoma) Desmin: Positive in rhabdomyosarcoma

 

Myoglobin: Positive in rhabdomyosarcoma

 

 

Epithelial membrane antigen: Positive in synovial sarcoma Correct Answer: S-100

 

3932. (3859) Q9-7642:

Which of the following is true concerning the surgical treatment of a schwannoma:

 

1) Radical margin is necessary

3) Marginal margin with sacrifice of nerve fibers as necessary

2) Wide margin is necessary

5) Intralesional or marginal margin with care to preserve all nerve fibers

4) Intralesional margin with sacrifice of nerve fibers as necessary

 

Schwannomas can be treated with observation or excision (intralesional or marginal). Every effort should be made to preserve all the nerve fibers.

Correct Answer: Intralesional or marginal margin with care to preserve all nerve fibers

 

3933. (3860) Q9-7643:

Patients with plexiform neurofibromas most likely have an abnormality in which of the following genes:

 

1) Runt related transcription factor 2 (Runx2)

3) Neurofibromatosis (NF)-1

2) Lipoprotein receptor-related protein 5 (LRP5)

5) Retinoblastoma gene (Rb-1)

4) NF-2

 

Patients with plexiform neurofibromas often have neurofibromatosis. The genetic abnormality is a mutation in the NF-1 gene. The other responses refer to:

 

 

Runx2: Crucial for bone formation, defect results in cleidocranial dysplasia LRP5: Deficiencies result in osteoporosis

 

NF-2: Mutations result in acoustic neuromas

 

 

Rb-1: Tumor-suppressor gene, defects make patients more prone to osteosarcoma and retinoblastoma Correct Answer: Neurofibromatosis (NF)-1

 

3934. (3861) Q9-7644:

Which of the following imaging features is characteristic of a neurofibroma:

 

1) Mineralized at the periphery and lucent in the center

3) Rim enhancement with contrast imaging

2) Fat suppression

5) Signal dropout on gradient echo sequences

4) T2-weighted peripheral high signal, low central signal

 

Neurofibromas often have a characteristic âtarget signâ on T2-weighted images. The target sign occurs because of peripheral high signal on T2-weighted images and low signal in the center.

The other responses refer to:

 

 

Heterotopic ossification: Mineralized at the periphery and lucent in the center Lipomas: Show complete fat suppression

 

Fluid-filled structures: Rim enhancement with contrast imaging

 

 

Pigmented villonodular synovitis: Signal dropout on gradient echo sequences Correct Answer: T2-weighted peripheral high signal, low central signal

3935. (3862) Q9-7645:

Which of the following describes the microscopic appearance of a neurofibroma:

 

1) Distinct hyoid and cellular areas

3) Spindle- and stellate-shaped cells in a mucoid matrix

2) Verocay bodies

5) Random bundles of coarse collagen and cells with wavy nuclei

4) Disorganized admixture of fibrous tissue and regenerative nerve sprouts

 

 

Neurofibromas have a characteristic appearance: Random bundles of collagen

 

 

Wavy nuclei Myxoid matrix

 

Other lesions that must be distinguished from neurofibromas include: Schwannoma: Myxoid and cellular areas

 

 

Antoni A: Cellular areas Antoni B: Myxoid areas

 

 

Verocay bodies: Parallel arrangement of nuclei Myxoma: Spindle- and stellate-shaped cells in a mucoid matrix

 

 

Neuroma: Disorganized admixture of fibrous tissue and regenerative nerve sprouts Correct Answer: Random bundles of coarse collagen and cells with wavy nuclei

 

3936. (3863) Q9-7646:

A patient has the following findings: axillary skin freckling, café au lait spots, multiple subcutaneous nodules, and Lisch nodules. Which of the following genetic abnormalities is most likely:

 

1) Runt related transcription factor 2 (Runx2)

3) Retinoblastoma gene (Rb-1)

2) Lipoprotein receptor-related protein 5 (LRP5)

5) NF-2

4) Neurofibromatosis (NF)-1

 

The described phenotype is most consistent with neurofibromatosis. The genetic abnormality is a mutation in NF-1. The other responses refer to:

 

 

Runx2: Crucial for bone formation, defect results in cleidocranial dysplasia LRP5: Deficiencies result in osteoporosis

 

NF-2: Mutations result in acoustic neuromas

 

 

Rb-1: Tumor-suppressor gene, defects make patients more prone to osteosarcoma and retinoblastoma Correct Answer: Neurofibromatosis (NF)-1

 

3937. (3864) Q9-7647:

Which of the following conditions has the highest risk of malignant change:

 

1) Neurofibroma

3) Plexiform neurofibroma

2) Schwannoma

5) Perineurioma

4) Myxoma within a nerve

 

Patients with plexiform neurofibromas often have neurofibromatosis. The risk of malignant change is approximately 5% to 10%. Plexiform neurofibroma has the greatest chance of malignant degeneration.

Correct Answer: Plexiform neurofibroma

 

3938. (2736) Q10-3233:

Which of the following molecules play(s) an important role influencing both embryonic bone formation and fracture healing:

 

1) Indian hedgehog (ihh) and core-binding factor 1 (cbf1)

3) Platelet-derived growth factor (PDGF)

2) Transforming growth factor beta (TGF-B)

5) Vascular endothelial growth factor (VEGF)

4) Interleukin-1 (IL-1)

 

Important concepts to remember in regard to signaling proteins include:

 

 

Indian hedgehog (ihh) and core-binding factor 1 (cbf1) influence both embryonic bone formation and fracture healing. Vascular endothelial growth factor (VEGF) plays a role in cartilage hypertrophy at the growth plate and during fracture healing.

 

Transforming growth factor beta (TGF-B) and platelet-derived growth factor (PDGF) are found in the early fracture hematoma and these factors modulate cell proliferation and differentiation.

 

Bone morphogenetic protein and interleukin 1 and 6 are expressed during cartilage formation.

Correct Answer: Indian hedgehog (ihh) and core-binding factor 1 (cbf1)

 

 

3939. (2737) Q10-3234:

Which of the following factors are present in the hematoma following a fracture and play a role in modulating cell proliferation and differentiation:

 

1) Indian hedgehog (ihh) and core-binding factor 1 (cbf1)

3) Interleukin-1 (IL-1)

2) Transforming growth factor beta (TGF-B) and platelet-derived growth factor (PDGF)

5) Vascular endothelial growth factor (VEGF)

4) Interleukin-6 (IL-6)

 

Important concepts to remember in regard to signaling proteins include:

 

 

Indian hedgehog (ihh) and core-binding factor 1 (cbf1) influence both embryonic bone formation and fracture healing. Vascular endothelial growth factor (VEGF) plays a role in cartilage hypertrophy at the growth plate and during fracture healing.

 

Transforming growth factor beta (TGF-B) and platelet-derived growth factor (PDGF) are found in the early fracture hematoma and these factors modulate cell proliferation and differentiation.

 

 

Bone morphogenetic protein and interleukin 1 and 6 are expressed during cartilage formation. Correct Answer: Transforming growth factor beta (TGF-B) and platelet-derived growth factor (PDGF)

 

3940. (2738) Q10-3235:

Which of the following factors play(s) an important role in cartilage hypertrophy during growth plate development and ossification in fracture healing:

 

1) Indian hedgehog (ihh) and core-binding factor 1 (cbf1)

3) Interleukin-1 (IL-1)

2) Transforming growth factor beta (TGF-B) and platelet-derived growth factor (PDGF)

5) Vascular endothelial growth factor (VEGF)

4) Interleukin-6 (IL-6)

 

Important concepts to remember in regard to signaling proteins include:

 

 

Indian hedgehog (ihh) and core-binding factor 1 (cbf1) influence both embryonic bone formation and fracture healing. Vascular endothelial growth factor (VEGF) plays a role in cartilage hypertrophy at the growth plate and during fracture healing.

 

Transforming growth factor beta (TGF-B) and platelet-derived growth factor (PDGF) are found in the early fracture hematoma and these factors modulate cell proliferation and differentiation.

 

Bone morphogenetic protein and interleukin 1 and 6 are expressed during cartilage formation.

Correct Answer: Vascular endothelial growth factor (VEGF)

 

3941. (2739) Q10-3236:

All of the following are important to achieve primary osteonal healing during plate fixation except:

 

1) Anatomic reduction

3) Adequate vascular supply

2) Rigid fixation

5) Very low strain levels

4) Moderate to high strain levels

 

In primary osteonal bond healing, osteoclasts cut channels across the bone contact sites and blood vessels and osteoblasts fill in the gap with new bone.

To achieve osteonal healing, there must be an anatomic reduction with rigid fixation. An adequate blood supply must be present and the amount of motion at the fracture must be very small to none (very low strain levels).

Moderate to high strain levels occur if there is motion at the osteosynthesis site. This motion can result from poor fixation (lack of rigidity) or excessive loading during the healing period. High strain levels in the gap favor the formation of granulation tissue rather than bone.

 

Correct Answer: Moderate to high strain levels

 

 

3942. (2740) Q10-3237:

All of the following factors will increase the rigidity of an external fixator except:

 

1) Increased individual pin diameter

3) Increased bone to rod distance

2) Increased pin number

5) Separating half pins by 45°

4) Increased pin group separation

 

 

Many factors influence the rigidity of an external fixator. Factors that increase the rigidity of the system include: Increased pin diameter

 

 

 

 

Increased pin number Decreased bone to rod distance Increased pin group separation Separating half pins by 45°

 

Increasing the bone to rod distance will decrease the rigidity of the system.

 

The fracture gap is also important. The gap should be minimized so that there is excellent bone apposition. Correct Answer: Increased bone to rod distance

3943. (2741) Q10-3238:

Which of the following graft types has both osteoinductive and osteoconductive properties:

 

1) Autogenous bone marrow

3) Bone morphogenetic protein-2

2) Coral-based hydroxyapatite bone graft substitute

5) Cancellous bone graft

4) Bone morphogenetic protein-7 (recombinant)

 

Grafting materials may have osteoconductive and/or osteoinductive, and osteoprogenitor cells.

Cancellous bone graft and vascularized bone grafts are the only materials that have significant osteoconductive, osteoinductive, and osteoprogenitor cells.

Several materials are mainly osteoinductive. Remember the definition of osteoinductive and osteoconductive properties:

 

Osteoinductive factors: molecules that have the capability of inducing osteoblastic precursors to differentiate into mature bone forming cells.

-Osteoinductive factors (without significant osteoconductive powers) include:

Growth factors:

BMP-2

rhBMP-7 (OP-1)

Demineralized bone matrix

 

Osteoconduction: the ability of a porous material to provide a scaffold for new bone formation.

-Osteoconductive materials without significant osteoinductive properties: Ceramics

Coral-based hydroxyapatite graft substitutes

Norian skeletal repair system (Norian Corporation, Cupertino, Calif)

Calcium sulfate pellets (Osteoset, Wright Medical Technology, Arlington, Tenn)

 

Bone marrow has the potential of supplying osteoprogenitor cells but has little osteoinductive or osteoconductive properties.

Correct Answer: Cancellous bone graft

 

3944. (2742) Q10-3239:

Which of the following materials has mainly osteoconductive properties with little or no osteoinductive ability:

 

1) Autogenous bone marrow

3) Bone morphogenetic protein-2

2) Coral-based hydroxyapatite bone graft substitute

5) Cancellous bone graft

4) Bone morphogenetic protein-7 (recombinant)

 

Grafting materials may have osteoconductive and/or osteoinductive, and osteoprogenitor cells.

Cancellous bone graft and vascularized bone grafts are the only materials that have significant osteoconductive, osteoinductive, and osteoprogenitor cells.

Several materials are mainly osteoinductive. Remember the definition of osteoinductive and osteoconductive properties:

 

Osteoinductive factors: molecules that have the capability of inducing osteoblastic precursors to differentiate into mature bone forming cells.

-Osteoinductive factors (without significant osteoconductive powers) include:

Growth factors:

BMP-2

rhBMP-7 (OP-1)

Demineralized bone matrix

 

Osteoconduction: the ability of a porous material to provide a scaffold for new bone formation.

-Osteoconductive materials without significant osteoinductive properties: Ceramics

Coral-based hydroxyapatite graft substitutes

Norian skeletal repair system (Norian Corporation, Cupertino, Calif)

Calcium sulfate pellets (Osteoset, Wright Medical Technology, Arlington, Tenn)

 

Bone marrow has the potential of supplying osteoprogenitor cells but has little osteoinductive or osteoconductive properties.

Correct Answer: Coral-based hydroxyapatite bone graft substitute

 

3945. (4056) Q10-3240:

Which of the following materials has mainly osteoinductive properties with little or no osteoconductive ability:

 

1) Autogenous bone marrow

3) Calcium sulfate crystals

2) Coral-based hydroxyapatite bone graft substitute

5) Cancellous bone graft

4) Bone morphogenetic protein-7 (recombinant)

 

Grafting materials may have osteoconductive and/or osteoinductive, and osteoprogenitor cells.

Cancellous bone graft and vascularized bone grafts are the only materials that have significant osteoconductive, osteoinductive, and osteoprogenitor cells.

Several materials are mainly osteoinductive. Remember the definition of osteoinductive and osteoconductive properties:

 

Osteoinductive factors: molecules that have the capability of inducing osteoblastic precursors to differentiate into mature bone forming cells.

-Osteoinductive factors (without significant osteoconductive powers) include:

Growth factors:

BMP-2

rhBMP-7 (OP-1)

Demineralized bone matrix

 

Osteoconduction: the ability of a porous material to provide a scaffold for new bone formation.

-Osteoconductive materials without significant osteoinductive properties: Ceramics

Coral-based hydroxyapatite graft substitutes

Norian skeletal repair system (Norian Corporation, Cupertino, Calif)

Calcium sulfate pellets (Osteoset, Wright Medical Technology, Arlington, Tenn)

 

Bone marrow has the potential of supplying osteoprogenitor cells but has little osteoinductive or osteoconductive properties.

Correct Answer: Bone morphogenetic protein-7 (recombinant)

 

 

3946. (2743) Q10-3241:

When electrically stimulated, bone tissue shows new bone formation at which of the following sites:

 

1) The positive electrode

3) The battery pack

2) The anode

5) The cathode

4) Bone marrow

 

Bone growth occurs at the negative electrode. The anode is a positive electrode, and the battery functions as the anode. Bone growth in bone marrow is not induced by electric stimulation.Correct Answer: The cathode

 

 

3947. (2744) Q10-3242:

Which of the following statements is true about electric bone physiology:

 

1) Bone tissue has no electric potential because it is predominantly made up of calcium and collagen.

3) Bone tissue has an electric potential that is dependent on the viability of the bone tissue.

2) Bone tissue has an electric potential that is dependent on the charge on the pain nerves.

5) Collagen contorts the electromagnetic field generated by muscles to produce a bone magnetic field.

4) Bone is electromagnetic by nature because calcium has a strong ferromagnetic effect.

 

Bone tissue has an electric potential that is dependent on its viability. Calcium and collagen have not been reported to be related to this field. Pain nerves do not generate the potential in bone. There is no bone magnetic field and collagen.Correct Answer: Bone tissue has an electric potential that is dependent on the viability of the bone tissue.

 

3948. (2745) Q10-3243:

All of the following are modalities of electrical stimulation used in practice except:

 

1) Direct current electric stimulation (DCES)

3) Pulsed electric magnetic stimulation (PEMF)

2) Capacitively coupled electric energy (CCES)

5) Combined magnetic fields (CMF)

4) Pulsed micromagnetic stimulation (PMMF)

 

There is no description of a pulsed micromagnetic stimulation (PMMF) device in scientific literature. All of the other choices are currently in practice.

Correct Answer: Pulsed micromagnetic stimulation (PMMF)

 

 

3949. (2746) Q10-3244:

Which of the following is the major disadvantage of using pulsed electric magnetic stimulation (PEMF):

 

1) PEMF must be implanted, thereby increasing the chance of infection.

3) PEMF can cause necrosis of surrounding tissue.

2) PEMF causes nausea and vomiting by stimulating the chemo trigger zone.

5) There are no disadvantages.

4) PEMF is dependent on patient compliance.

 

Patient compliance is a drawback to the use of PEMF. A direct current electric stimulation device must be implanted, whereas PEMF is usually part of a brace. None of the devices has been described to cause nausea.Correct Answer: PEMF is dependent on patient compliance.

 

 

3950. (2747) Q10-3245:

Which of the following statements is true:

 

1) Electromagnetic stimulation has not been shown clinically effective.

3) Pulsed electric magnetic stimulation (PEMF) is the only FDA-approved modality as an adjunct to spinal fusion.

2) Pending trials in primates, electromagnetic stimulation is not yet a U.S. Food and Drug Adminstration (FDA)-approved modality.

5) The FDA has banned electromagnetic stimulation because it causes serious cardiac arrhythmias.

4) Electromagnetic stimulation appears to be effective in spinal fusion, but it is not widely used.

 

Electromagnetic stimulation is FDA approved and has been shown to be clinically effective in more than one study. Although effective, it is not commonly used during routine fusions.

Correct Answer: Electromagnetic stimulation appears to be effective in spinal fusion, but it is not widely used.

 

 

3951. (2748) Q10-3246:

In terms of the osteoinductive capacity of different demineralized bone matrix (DBM) products, which of the following statements is true:

 

1) Because the bone morphogenetic protein (BMP) content of DBM is regulated by the U.S. Food and Drug Administration (FDA), osteoinductive capacity is equal among DBM products.

3) The osteoinductive capacity of DBM is not affected by the method of sterilization.

2) The osteoinductive capacity of DBM is unaffected by the age of the donor because the calcium is removed during processing.

5) The osteoinductive capacity of DBM is affected by age of the donor and method of sterilization.

4) Overall efficacy is equal because each batch of DBM is made from a pool of donors that minimizes variation.

 

The osteoinductive capacity of DBM can be affected by the method of processing, storage, sterilization, and the age of the donor. Although calcium is removed during the demineralization process, this does not affect the osteoinductivity of DBM. The FDA requires that each batch of DBM be obtained from a single human donor, not from a pool of donors. The content of BMP in DBM is not regulated by the FDA.Correct Answer: The osteoinductive capacity of DBM is affected by age of the donor and method of sterilization.

 

3952. (2749) Q10-3247:

Demineralized bone matrix (DBM) regenerates bone through which of the following mechanisms:

 

1) Osteogenesis

3) Osteoinduction

2) Osteoconduction

5) Osteoconduction and osteoinduction

4) Osteogenesis and osteoinduction

 

Demineralized bone matrix acts as an osteoconductive and possible osteoinductive material. The osteoinductive capability of DBM is attributed to the bone morphogenetic proteins (BMPs) present in the extracellular matrix of the donor bone, which are preserved during the demineralization process. However, the amount of BMP within DBM is smaller than the supraphysiologic amounts provided with recombinant human BMP. Osteogenesis refers to a bone graft that supplies live osteoblasts or stem cells that can differentiate into osteoblasts. Only autogenous bone grafts and bone marrow aspirates are considered osteogenic.Correct Answer: Osteoconduction and osteoinduction

 

 

3953. (2750) Q10-3248:

Demineralized bone matrix (DBM) is indicated for which of the following situations:

 

1) Unstable, contained skeletal defects

3) As a supplement when an adequate autogenous bone graft is also used

2) Stable, contained skeletal defects

5) Any circumstance where autogenous bone grafts are recommended

4) Fractures requiring additional structural support after internal fixation

 

Because DBM does not provide structural support, recommended uses include stable contained skeletal defects, such as bone cysts and cavities. Although many authors have reported healing rates similar to autogenous bone grafting historical controls, no randomized studies offer a true comparison of DBM to autogenous bone grafting. Additional uses of DBM include augmentation of autogenous bone grafts when the supply of graft is limited or the defect is large, but it is not used when there is adequate autogenous bone graft.Correct Answer: Stable, contained skeletal defects

 

 

3954. (2751) Q10-3249:

Demineralized bone matrix (DBM) contains which of the following substances:

 

1) Collagen

3) Bone growth factors such as bone morphogenetic protein (BMP)

2) Noncollagenous proteins

5) Collagen, Noncollagenous proteins, and bone growth factors such as bone morphogenetic protein (BMP)

4) Collagen and bone growth factors such as bone morphogenetic protein (BMP)

 

Demineralized bone matrix (DBM) is formed by means of acid extraction from cadaver bone, leaving noncollagenous proteins, collagen, and a small amount of bone growth factors, such as BMP.Correct Answer: Collagen, Noncollagenous proteins, and bone growth factors such as bone morphogenetic protein (BMP)

 

 

3955. (2752) Q10-3250:

When reviewing the marketing claims of various demineralized bone matrix (DBM) products, which of the following statements is true:

 

1) Claims are accurate because they are monitored by the U.S. Food and Drug Administration.

3) Claims are usually inaccurate because studies show no difference among various companiesâ DBM.

2) Claims are usually based on published, peer-reviewed research.

5) Claims are true if they refer to DBM alone, rather than its combination with other bone graft substitutes.

4) Claims should be interpreted with caution.

 

The claims made by various companies and tissue banks concerning their DBM should be interpreted with caution. Although there are variations in processing, there are usually little comparative scientific data available to support companiesâ claims that their processing method offers unique advantages and superior performance over other products. In the United States, DBM is classified as a minimally manipulated tissue for transplantation. Historically, there has been little regulatory control over the claims made by tissue processing facilities.Correct Answer: Claims should be interpreted with caution.

 

3956. (2811) Q10-3312:

Calcium phosphate is a bone graft substitute that can be best described as:

 

1) Osteogenic

3) Osteoinductive

2) Osteoconductive

5) None of the above

4) Both osteoconductive and osteoinductive

 

Calcium phosphate is an osteoconductive bone graft substitute. Over time, calcium phosphate undergoes osteoclastic resorption followed by the invasion of small blood vessels that become surrounded by circumferential lamellae of new bone. Calcium phosphate does not have any osteogenic or osteoinductive capacity.Correct Answer: Osteoconductive

 

 

3957. (2812) Q10-3313:

Compared with cancellous bone and other bone graft substitutes, the compressive strength of calcium phosphate is:

 

1) Less than cancellous bone and less than other bone graft substitutes

3) Less than cancellous bone but greater than other bone graft substitutes

2) Greater than cancellous bone but less than other bone graft substitutes

5) Equal to cancellous bone and other bone graft substitutes

4) Greater than both cancellous bone and other bone graft substitutes

 

Compared with cancellous bone grafts and other bone graft substitutes, calcium phosphate has a high compressive strength. For example, once Norian SRS (Skeletal Repair System, Synthes, Paoli, Pa) hardens, approximately 12 hours after injection, its compressive strength is approximately 55 MPa. This is roughly 4 to 10 times greater than the average compressive strength of cancellous bone.Correct Answer: Greater than both cancellous bone and other bone graft substitutes

 

 

3958. (2813) Q10-3314:

Calcium phosphate provides which of the following advantages in fracture repair:

 

1) Improved rate of fracture healing

3) Supports internal fixation and resists redisplacement of fracture fragments

2) Eliminates the need for use of metallic internal fixation implants

5) Supports internal fixation and resists redisplacement of fracture fragments and may permit earlier weight bearing or functional use

4) May permit earlier weight bearing or functional use

 

Numerous biomechanical studies have shown that calcium phosphate increases internal fixation strength and stiffness, and resists redisplacement of reduced fracture fragments.

Calcium phosphate is intended to be used in conjunction with some form of internal fixation. In a biomechanical study of simulated distal radius fractures, calcium phosphate was found to be insufficient to withstand the physiologic loads associated with wrist flexion and extension unless supplemental K-wire fixation was added.

 

Several clinical studies have reported good success when fracture fixation was augmented with calcium phosphate. This allowed early weight bearing and functional use of the extremity with good results.

Calcium phosphate is an osteoconductive bone graft substitute. It does not increase the speed of healing. However, preclinical studies have shown its effectiveness as a carrier for rhBMP-2. This provided gradual release of rhBMP-2 and accelerated bone repair.

 

Correct Answer: Supports internal fixation and resists redisplacement of fracture fragments and may permit earlier weight bearing or functional use

 

3959. (2814) Q10-3315:

When calcium phospate (eg. Norian SRS (Skeletal Repair System, Synthes, Paoli, Pa) hardens in situ and cures by crystallization, it forms what structure that is similar to that found in the mineral phase of bone:

 

1) Tri-calcium phosphate

3) Calcium

2) Dahllite

5) Collagen

4) Mineralized cartilage

 

Calcium phosphate hardens in situ and cures by a crystallization reaction to form dahllite, a carbonated apatite that is similar to that found in the mineral phase of bone.Correct Answer: Dahllite

 

 

3960. (2815) Q10-3316:

When extrusion of calcium phosphate into the joint occurs following its percutaneous injection in intra-articular fractures, which of the following complications has been reported:

 

1) Joint ankylosis

3) Infection

2) Rapid, progressive degenerative changes

5) None of the above

4) Inflammation with surrounding soft tissue pain

 

Although intra-articular extrusion of calcium phosphate should be avoided, when it has occurred there have been no reported adverse sequelae.

Painful soft tissue response was reported in 4 of 20 patients who underwent curettage of enchondromas and injection with Norian SRS (Skeletal Repair System, Synthes, Paoli, Pa). This prompted the authors to discontinue this method of enchondroma treatment and recommend that the cement be "sealed" by placing curetted bone over the exposed surface of the cement.

 

Correct Answer: None of the above

 

 

3961. (2918) Q10-3419:

What are the desirable osteogenic cells to exploit from bone marrow aspirates:

 

1) Differentiated mesenchymal cells and granulocytes

3) Granulocytes and fibroblasts

2) Undifferentiated mesenchymal cells and osteoprogenitor cells

5) Granulocytes and erythrocytes

4) Erythrocytes and osteoprogenitor cells

 

Undifferentiated mesenchymal stem cells have the ability to proceed down any cell lineage and osteoprogenitor cells are prepared to proceed down bone-forming lineages. Both cell lines are crucial in the production of new bone.Correct Answer: Undifferentiated mesenchymal cells and osteoprogenitor cells

 

 

3962. (2919) Q10-3420:

Bone marrow aspirates are generally acquired in what incremental volumes to maximize efficient yield:

 

1) 1 cc

3) 3 cc

2) 2 cc

5) 5 cc

4) 4 cc

 

Harvesting aspirates in no greater than 2 mL aliquots leads to the most efficient yield. Beyond this, one begins to draw venous backflow of blood.Correct Answer: 2 cc

 

3963. (2920) Q10-3421:

All of the following factors have been implicated in effecting cell composition of marrow-derived cells except:

 

1) Donor age

3) Donor ethnicity

2) Osteoporosis

5) Donor gender

4) Aspiration volume

 

Wide variability in marrow-derived cells was shown among individual patients and was attributed to differences in donor age, gender, aspiration volume, and osteoporosis. Although donor ethnicity has been suggested, no studies have established this association.Correct Answer: Donor ethnicity

 

 

3964. (2921) Q10-3422:

All of the following are mechanisms that have been investigated in preparation and concentration of bone marrow aspirate except:

 

1) Ionic separation

3) Centrifugation

2) Vortex

5) Sedimentation

4) Diffusion chamber

 

Marrow osteoprogenitor cells are aspirated from the iliac crest and injected into the desired surgical site to effectively stimulate osteogenesis. Before application, the aspirate can be prepared and concentrated through a variety of methods including vortex, centrifugation, sedimentation, and diffusion chamber. Ionic separation has not been used as a mechanism to prepare or concentrate bone marrow.Correct Answer: Ionic separation

 

 

3965. (2922) Q10-3423:

Which of the following has been shown to be a significant difficulty associated with osteoprogenitor cell cloning:

 

1) Cloned cells produce mature osseous tissue later than mixed marrow cell aspirate.

3) Cloned cells often contain contaminants that significantly increase post-implantation morbidity.

2) Using cells obtained from cloning for spinal fusion does not increase fusion rates above those of mixed marrow cells.

5) Cloned osteoprogenitor cells have a tendency to terminally differentiate resulting in a reduced concentration of cells capable of osteogenesis

4) Cloned osteoprogenitor cells do not produce mature osseous tissue to a greater degree than mixed marrow cell aspirate.

 

Cloned osteoprogenitor cells have been shown to produce mature osseous tissue earlier and to a greater degree than mixed marrow cell aspirate. Fusion rates are significantly increased with cloned osteoprogenitor cells as compared to mixed marrow cells.

 

The difficulty with the osteoprogenitor cloning technique, however, is that the cells have a tendency to terminally differentiate as they are clonally expanded, resulting in a decreased concentration of cells capable of osteogenesis. Moreover, establishing cells as belonging to an osteoprogenitor lineage can be challenging.

 

Cloned cells have not been shown to contain contaminants or increase morbidity.

Correct Answer: Cloned osteoprogenitor cells have a tendency to terminally differentiate resulting in a reduced concentration of cells capable of osteogenesis

 

3966. (2923) Q10-3424:

Clinically used structural allografts may be:

 

1) Osteoconductive

3) Osteogenic

2) Osteoinductive

5) Osteoconductive, Osteoinductive and Osteogenic

4) Osteoconductive and Osteoinductive

 

Allograft osteogenic characteristics are lost during tissue processing for transplantation.

 

 

Allograft contains structural collagen for new bone deposition (osteoconduction). Allograft is not osteogenic.

Correct Answer: Osteoconductive

 

 

3967. (2924) Q10-3425:

Fresh frozen allograft is prepared by:

 

1) Taking from the cadaver for direct transplantation

3) Treating with acid to extract a composite of collagen, noncollagenous proteins, and a low concentration of growth factors

2) Washing in an antibiotic solution, cooled to -70º C, and dehydrated through a vacuum process to reduce the water content to less than 5%

5) Washing in an antibiotic solution, treated with acid, and cooled to -70º C

4) Washing in an antibiotic solution and cooled to -70º C

 

 

Fresh frozen allograft is frozen after its harvest; after being washed in an antibiotic solution, it is cooled to -70º C. Direct transplantation describes fresh preparation.

 

 

Freeze-dried allograft undergoes a process called lyophilization, where the specimen is washed, cooled, and dehydrated. Acid treatment describes the process of demineralization.

 

Fresh frozen specimens are not treated with acid.

Correct Answer: Washing in an antibiotic solution and cooled to -70º C

 

 

3968. (2925) Q10-3426:

While discussing surgical options regarding her upcoming spinal fusion, a patient asks about the risks of human immunodeficiency virus (HIV) transmission in allograft use. Which of the following is the best response:

 

1) There has never been a reported case of HIV transmission from allograft.

3) The estimated risk of HIV transmission from allograft is approximately 1 in 1 trillion.

2) HIV cannot survive in tissue once it is removed from the host.

5) The estimated risk of HIV transmission from allograft is approximately 1 in 100.

4) The estimated risk of HIV transmission from allograft is approximately 1 in 1 million.

 

 

Fresh frozen allograft is frozen after its harvest; after being washed in an antibiotic solution, it is cooled to -70º C. HIV has been transmitted to a reported 4 cases of fresh frozen allograft.

 

HIV can survive in blood and tissue specimens outside the body.

 

Allograft carries the risk of HIV transmission in less than 1 per 1 million uses.

Correct Answer: The estimated risk of HIV transmission from allograft is approximately 1 in 1 million.

 

3969. (4057) Q10-3427:

Which of the following statements is true regarding posterior allograft applications:

 

1) In the adult population, allograft fusions perform better under conditions of tension rather than compression.

3) Allograft is far superior to autograft in posterior applications for spinal fusion.

2) Allograft fusions have shown success in pediatric patients with scoliosis.

5) Radiographic studies examining side-by-side comparison of autograft and allograft show similar size fusion masses.

4) In comparison to allograft, procedures using autograft have less blood loss and operating time.

 

The pediatric population has been the exception to the poor results of allograft in posterior application.

 

 

Allograft performs better under conditions of compression than tension. Autograft is superior to allograft in posterior applications.

 

 

Procedures using allograft have less blood loss and shorter operating time. Radiographic studies of side-by-side comparisons show smaller allograft fusion masses.

Correct Answer: Allograft fusions have shown success in pediatric patients with scoliosis.

 

 

3970. (2926) Q10-3428:

Which statement is not true concerning anterior applications of allograft in the spine:

 

1) Smoking is a risk factor for developing nonunion in allograft fusions

3) Self-assessment questionnaires have shown greater improvement in function, less pain, and less medication usage in patients receiving an allograft.

2) In the adult population, allograft fusions perform better under conditions of compression rather than tension.

5) Allograft fusion masses are larger than autograft fusion masses.

4) Allograft has better success than autograft in multilevel fusion studies.

 

Many studies have shown higher fusion rates for autograft vs. allograft in multilevel studies.

 

Smoking is a major risk factor for nonunion.

 

Allograft fuses better under compression forces than tension forces.

 

Questionnaires reveal that patients who receive an allograft show improvement in function, less pain, and less medication usage.

Correct Answer: Allograft has better success than autograft in multilevel fusion studies.

 

 

3971. (2927) Q10-3429:

Calcium sulfate is a bone graft substitute that can be best described as:

 

1) Osteogenic

3) Osteoinductive

2) Osteoconductive

5) Both osteoinductive and osteogenic

4) Both osteoconductive and osteogenic

 

Most authors describe calcium sulfate as an osteoconductive bone graft substitute. Calcium sulfate is relatively rapidly resorbed in 4 to 12 weeks and provides only short-term three-dimensional framework to support osteoconduction.Correct Answer: Osteoconductive

 

3972. (2928) Q10-3430:

The compressive strength of calcium sulfate bone graft substitute pellets is:

 

1) Greater than calcium phosphate

3) Similar to cancellous bone

2) Much less than cancellous bone

5) Not as great as its bending strength

4) Not as great as its tensile strength

 

Calcium sulfate has a compressive strength equal to or greater than cancellous bone grafts. The compressive strength of calcium sulfate is much less than the compressive strength of calcium phosphate, which once cured has a compressive strength of approximately 55 MPa (roughly four to 10 times greater than the average compressive strength of cancellous bone). Because of its low compressive strength, calcium sulfate is indicated only for use in bone defects that are not intrinsic to the stability of the bone structure. Calcium sulfate has no appreciable tensile, bending, or torsional strength.Correct Answer: Similar to cancellous bone

 

 

3973. (2929) Q10-3431:

The addition of antibiotics to calcium sulfate bone graft substitutes can best be described as:

 

1) Ineffective because the antibiotics are inactivated when they become bound to the calcium sulfate

3) Less effective than debridement alone when examined in a rabbit osteomyelitis model

2) Limited because tobramycin cannot be used with calcium sulfate

5) Effective in animal models and effective in human clinical study

4) Effective in animal models, but not effective in human clinical study

 

The relatively rapid resorption of calcium sulfate makes it an ideal carrier for gradual release of antibiotics. Calcium sulfate impregnated with tobramycin was found to be more effective at eradication of infection than either debridement alone or treatment with intramuscular antibiotics in a rabbit osteomyelitis model. In a preliminary published report of its use in patients with infected long bone defects and nonunions, successful eradication of infection was achieved in 23 of 25 patients.Correct Answer: Effective in animal models and effective in human clinical study

 

 

3974. (2930) Q10-3433:

A common complication associated with the use of calcium sulfate bone graft substitutes includes:

 

1) Sterile abscess

3) Serous wound drainage

2) Hypercalcemia

5) Aggressive foreign body giant cell reaction

4) Malignant transformation

 

Wound drainage was reported in 4% of patients in a series of 109 patients treated with calcium sulfate pellets. Researchers theorize that this serous drainage is due to the osmotic effect of the calcium sulfate and subsides with the eventual resorption of the calcium sulfate. Calcium sulfate produces little or no foreign bone giant cell reaction.Correct Answer: Serous wound drainage

 

 

3975. (2960) Q10-3466:

Coralline hydroxyapatite is a bone graft substitute that can be best described as:

 

1) Osteogenic

3) Osteoinductive

2) Osteoconductive

5) None of the above

4) Both osteoconductive and osteoinductive

 

Coralline hydroxyapatite is an osteoconductive bone graft substitute. It has a 3-dimensional structure of interconnected porosity that mimics cancellous bone. Capillaries, perivascular tissue, and osteoprogenitor cells migrate into these porous spaces and form bone.Correct Answer: Osteoconductive

 

3976. (2961) Q10-3467:

Compared with cancellous bone, the compressive strength of coralline hydroxyapatite is:

 

1) Greater than cancellous bone

3) Greater than cancellous bone but decreases with tissue ingrowth

2) Greater than cancellous bone and increases with tissue ingrowth

5) Similar to cancellous bone but decreases with tissue ingrowth

4) Similar to cancellous bone and increases with tissue ingrowth

 

The compressive strength of coralline hydroxyapatite is similar to cancellous bone. Its compressive strength increases as the tissue ingrowth is converted to mature lamellar bone.Correct Answer: Similar to cancellous bone and increases with tissue ingrowth

 

 

3977. (2962) Q10-3468:

Pro-osteon(Interpore) bone graft substitute is produced from marine coral using a process that converts calcium carbonate into what substance:

 

1) Cancellous bone

3) hydroxyapatite

2) Calcium sulfate

5) Carbonic acid

4) Beta tricalcium phosphate

 

Coralline hydroxyapatite is produced from marine coral exoskeletons that have a regular pore structure, resembling the structure of cancellous bone. The genus Gonipora has a microstructure similar to that of cancellous bone, with parallel longitudinal pores of 600 µm, with interconnecting channels ranging from 220 µm to 260 µm. Optimal pore size and interconnectivity are important for vascular ingrowth. The coral is subjected to high pressure and heated in an aqueous phosphate solution, converting the calcium carbonate coral skeleton into hydroxyapatite [C 1 0 (PO4 )6 (OH)2 ].Correct Answer: hydroxyapatite

 

 

 

3978. (2963) Q10-3469:

Which of the following statements is true concerning the use of coralline hydroxyapatite in the management of tibial plateau fractures:

 

1) Coralline hydroxyapatite has sufficient strength to buttress the reduction as long as patients remain non-weight bearing.

3) Coralline hydroxyapatite requires internal fixation to maintain reduction.

2) Coralline hydroxyapatite has sufficient strength to buttress the reduction even with weight bearing.

5) Coralline hydroxyapatite has little chance for bone ingrowth due to the fracture location.

4) Coralline hydroxyapatite is inferior to autogenous bone graft.

 

Coralline hydroxyapatite does not have sufficient mechanical strength to be used alone and internal fixation is required. In a randomized study of patients treated with either autogenous bone graft or coralline hydroxyapatite, there were no significant differences in either the radiologic or clinical assessments. Histologic analysis of human and animal biopsy specimens show bone ingrowth into the porous implant.Correct Answer: Coralline hydroxyapatite requires internal fixation to maintain reduction.

 

 

3979. (2964) Q10-3470:

What is considered the main property of coralline hydroxyapatite that makes it a successful bone graft substitute:

 

1) Presence of calcium phosphate for binding of osteoblasts

3) 3-dimensional interconnected pore structure

2) Presence of calcium carbonate for binding of osteoblasts

5) Presence of calcium carbonate for binding of osteocytes

4) Presence of calcium phosphate for binding of osteocytes

 

The 3-dimensional structure of coralline hydroxyapatite with its interconnected porosity that mimics cancellous bone is considered to be the main property that makes it a successful bone graft substitute. This scaffold allows invasion of capillaries, perivascular tissue, and osteoprogenitor cells with the subsequent formation of bone.Correct Answer: 3-dimensional interconnected pore structure

 

3980. (2986) Q10-3492:

Collagen-based matrices, such as Collagraft (Zimmer, Warsaw, Ind) or Healos (DePuy Acromed, Raynham, Mass), are indicated for use in which manner?

 

1) In areas requiring a mechanically strong bone graft substitute

3) In conjunction with antibiotics for the treatment of osteomyelitis

2) In articular fractures with cartilage defects

5) In conjunction with demineralized bone matrix

4) In conjunction with autogenous bone marrow

 

Collagen-based matrices are indicated for use in conjunction with autogenous bone marrow. The collagen matrix provides an osteoconductive scaffold. Addition of autogenous bone marrow provides osteogenic cells. It is not indicated for use in osteomyelitis or in conjunction with demineralized bone matrix. The material has no mechanical strength and so must be used in conjunction with internal or external fixation. It is not indicated for the repair of intra-articular cartilage defects.Correct Answer: In conjunction with autogenous bone marrow

 

 

3981. (2987) Q10-3493:

Contraindications for the use of collagen-based matrices include all of the following except:

 

1) Patients with a known hypersensitivity to bovine collagen

3) Patients with multiple severe allergies

2) Patients with a history of anaphylaxis

5) Patients with rheumatoid arthritis

4) Patients undergoing desensitization injections to meat products

 

Collagen-based matrices should not be used in patients with severe allergies (eg, history of anaphylaxis, history of multiple severe allergies) or known allergies to bovine collagen. It should also not be used in patients who are undergoing desensitization injections to meat products because these may contain bovine collagen. The use of xenographic collagen has raised concern of the risk that its immunogenicity may elicit a harmful immune reaction. In the collagraft clinical study, a small percentage of patients developed antibodies to type I bovine collagen, but these did not show any cross-reactivity to human collagen and did not result in dermal hypersensitivity and did not appear to affect function of the bone graft substitute.Correct Answer: Patients with rheumatoid arthritis

 

 

 

3982. (2988) Q10-3494:

Collagen-based matrices are composed of which of the following substances:

 

1) Porcine type I collagen

3) Bovine type I and III collagen

2) Porcine type I and III collagen

5) Recombinant human collagen proteins

4) Bovine Type I and VI collagen

 

 

 

Collagen-based matrices are prepared from bovine dermis. They contain primarily type I collagen (>95%) and a small amount of type III collagen (<5%). Collagraft (Zimmer, Warsaw, Ind) consists of this collagen matrix combined with biphasic calcium

phosphate ceramic granules (65% hydroxyapatite and 35% -tricalcium phosphate). Healos (DePuy Acromed, Raynham, Mass) consists of a matrix of cross-linked bovine type-I collagen fibers that are circumferentially coated with hydroxyapatite using a proprietary 360° accretion process. Both products are intended to be used with the addition of autogenous bone marrow.Correct Answer: Bovine type I and III collagen

 

3983. (2989) Q10-3495:

In the absence of autogenous bone marrow, collagen-based matrices such as Collagraft (Zimmer, Warsaw, Ind) or Healos (DePuy Acromed, Raynham, Mass) can best be described as:

 

1) Osteogenic

3) Osteoinductive

2) Osteoconductive

5) Osteogenic and osteoinductive

4) Osteoconductive and osteoinductive

 

Collagen-based matrices act primarily as osteoconductive bone graft substitutes. The addition of autogenous bone marrow adds live osteoblasts and stem cells that are osteogenic. While the product itself is not osteoinductive, osteoinductive properties have been shown when combined with autogenous bone marrow in a rat model. Other authors question whether this combination of materials is osteoinductive.Correct Answer: Osteoconductive

 

 

3984. (2990) Q10-3496:

When compared with autogenous bone graft in the management of acute fractures, collagraft has been shown to be:

 

1) Less effective than autogenous bone graft

3) As effective and with an equal morbidity to autogenous bone graft

2) As effective but with a higher risk due to allergic reactions

5) Unable to determine because of inadequate follow-up in clinical studies

4) As effective and with a lower morbidity than autogenous bone graft

 

Chapman and colleagues performed a prospective, randomized, multicenter study to compare autogenous bone graft to Collagraft with autogenous bone marrow in long bone fractures. Inclusion criteria were patients between 18 and 70 years old with a fracture of the ulna, radius, humerus, tibia, or femur that was being treated with internal or external fixation and bone grafting within 30 days of their initial injury. The criteria for requiring bone grafting were based on the judgment of the operating surgeon. In this study, they followed 213 patients with 249 fractures for a minimum of 24 months to monitor fracture healing and the occurrence of complications. They found no significant differences in fracture healing rates (Collagraft 98% at 24 months compared to 99% for autogenous bone graft) or complication rates. The authors concluded that for traumatic defects of long bones requiring grafting, Collagraft was justified based on its safety, effectiveness, and decreased operative time and risk from eliminating the autogenous iliac crest bone graft. Preliminary findings of this multicenter study were also published in 1991. Autogenous bone grafting is associated with a known morbidity. Younger and Chapman reported an overall major complication rate of 8.6% and a minor complication rate of 20.6%.

 

Major criticism of the Collagraft clinical trial is the absence of two appropriate control groups (no additional bone graft substitute, autogenous bone marrow alone). It is argued that many of these fractures may have healed without the addition of a bone graft substitute, and this may be especially true with the current emphasis for indirect fracture reduction to optimize preservation of fracture site blood supply.

 

Correct Answer: As effective and with a lower morbidity than autogenous bone graft

 

 

3985. (3088) Q10-3597:

Compared with human bone, the compressive strength of tricalcium phosphate is:

 

1) Greater than cortical bone

3) Similar to cancellous bone

2) Greater than cancellous bone

5) Because it is a bone graft substitute, its mechanical properties cannot be compared to human bone.

4) Less than cancellous bone

 

The compressive strength of tricalcium phosphate is similar to that of cancellous bone. The material is resorbed relatively rapidly, weakening its compressive strength. Therefore, tricalcium phosphate is not recommended for use without internal fixation or in areas of stress intrinsic to the bone stability. Of the many bone graft substitutes, only calcium phosphate has a compressive strength that is greater than cancellous bone.

 

Correct Answer: Similar to cancellous bone

 

3986. (3089) Q10-3598:

Compared to the mineral composition of bone, tricalcium phosphate is characterized as:

 

1) Having a significantly greater concentration of calcium

3) Having a similar concentration of calcium and phosphate

2) Having a significantly greater concentration of phosphate

5) Having a significantly lower concentration of phosphate

4) Having a significantly lower concentration of calcium

 

Tricalcium phosphate [Ca3 (PO4 )2 ] bone graft substitutes are composed of 39% calcium and 20% phosphorus by weight in a molar concentration of 1.5. This composition is similar to the mineral content of bone (35% calcium, 15% phosphorus).

Correct Answer: Having a similar concentration of calcium and phosphate

 

 

3987. (3090) Q10-3599:

Indications for the use of tricalcium phosphate include:

 

1) Critical size segmental bone defects

3) Physeal fractures in children

2) Infected nonunion unions, where it can be used in combination with antibitoics

5) Defects that are not intrinsic to the stability of bone structure

4) Fractures in which mechanical stabilization cannot be obtained

 

The use of a tricalcium phosphate bone graft substitute is indicated for defects that are not intrinsic to the stability of the bone structure. Tricalcium phosphate does not provide sufficient mechanical support to be used without adjunctive internal or external fixation. Its use is contraindicated in segmental defects, infection, and physeal fractures.

 

Correct Answer: Defects that are not intrinsic to the stability of bone structure

 

 

3988. (3091) Q10-3600:

Which of the following statements is true concerning the resorption of tricalcium phosphate in humans:

 

1) Occurs through an inflammatory giant cell response

3) Occurs gradually over a period of 6 to 18 months

2) Occurs rapidly within 6 weeks of placement

5) The volume of bone ingrowth usually exceeds the volume of tricalcium phosphate that is resorbed.

4) Because its composition is similar to that of bone, the rate of resorption is extremely slow and occurs only after many years.

 

Tricalcium phosphate undergoes resorption via dissolution and fragmentation over a period of 6 to 18 months. Resorption of tricalcium phosphate occurs by osteoclasts without an inflammatory or giant cell response. The bone volume produced is always less than the volume of the tricalcium phosphate that is resorbed.

 

Compared to other osteoconductive bone graft substitutes, the resorption rate of tricalcium phosphate is slower than calcium sulfate, but much more rapid than hydroxyapatite.

Correct Answer: Occurs gradually over a period of 6 to 18 months

 

3989. (3435) Q10-4356:

Teriparatide (Forteo, Eli Lilly) is an injectable recombinant human parathyroid hormone (rhPTH) approved in the United States:

 

1) For use in the treatment of postmenopausal women with osteoporosis

3) To enhance fracture repair

2) To increase bone mass in men with primary hypogonadal osteoporosis

5) For use in the treatment of postmenopausal women with osteoporosis, to increase bone mass in men with primary hypogonadal osteoporosis, and to enhance fracture repair

4) For use in the treatment of postmenopausal women with osteoporosis, and to increase bone mass in men with primary hypogonadal osteoporosis

 

Teriparatide (Forteo, Eli Lilly) is an injectable rhPTH for the treatment of postmenopausal women with osteoporosis who are at risk for fracture. It is also indicated to increase bone mass in men with primary hypogonadal osteoporosis who are at high risk for fracture. Teriparatide is administered as a subcutaneous injection into the thigh or abdominal wall, once daily, at a recommended dosage of 20 mg. Because the safety and efficacy of teriparatide has not been evaluated beyond 2 years of treatment, its use for longer than 2 years is not recommended.

 

The use a teriparatide for enhancing fracture healing is not a current indication. There are ongoing clinical trials evaluating the effect of teriparatide on fracture healing.

Correct Answer: For use in the treatment of postmenopausal women with osteoporosis, and to increase bone mass in men with primary hypogonadal osteoporosis

 

 

3990. (3436) Q10-4359:

Current phase 2 clinical studies are evaluating the systemic administration of what agent as a possible means of enhancing fracture healing:

 

1) Recombinant human growth hormone

3) HMG-CoA reductase inhibitors (statins)

2) Recombinant human parathyroid hormone

5) Calcium sulfate

4) Calcium gluconate

 

A phase 2 (clinical study designed to evaluate product safety) multicenter international study evaluating the effects of teriparatide on distal radius fracture healing is in progress. A total of 105 patients are expected to participate in this study. The primary end point is the time to radiographic healing.

 

Preclinical evidence suggests that certain lipid-lowering drugs (HMG-CoA reductase inhibitors) may play a role in blocking bone catabolism, and some studies indicate these drugs increase bone mineral density and reduce the risk of fracture; other studies have challenged these findings.

 

Correct Answer: Recombinant human parathyroid hormone

 

 

3991. (3437) Q10-4362:

The prevalence of mesenchymal stem cells in human bone marrow aspirates is:

 

1) 1,000 per 100,000 nucleated cells

3) Not dependent on patient age

2) 1 per 100,000 nucleated cells

5) Independent of aspiration volume

4) Nearly constant between people of the same age

 

While bone marrow aspirates contain mesenchymal stem cells, the number and concentration of stem cells is limited. Only 1 per 100,000 nucleated bone marrow cells is a mesenchymal stem cell. The number of stem cells declines with patient age. In addition, variability exists between patients of the same age. Studies have shown that large volume bone marrow aspiration dilutes the number of stem cells with peripheral blood.Correct Answer: 1 per 100,000 nucleated cells

 

3992. (3438) Q10-4363:

In a clinical study in which bone marrow concentrate was percutaneously injected into noninfected tibial nonunions, the success of treatment was found to be dependent on:

 

1) Duration of the nonunion

3) Number and concentration of stem cells implanted

2) Volume of cells injected

5) Associated comorbidities

4) Age of the patient

 

Recently, Hernigou and colleagues reported on the use of percutaneous autologous marrow grafting in the management of 60 noninfected tibial nonunions. Twelve patients had initially been treated in a cast, while the other 48 were initially open fractures treated with external fixation. The definition of nonunion was failure to heal within 6 months of the injury, with no progressive radiographic healing between 3 and 6 months after injury. They aspirated bone marrow from the iliac crest, filtered it to separate cellular aggregate and fat, and then centrifuged it for 5 minutes, reducing a 300-cc volume of bone marrow aspirate to 50 cc. The concentrated bone marrow aspirate was than percutaneously injected in the nonunion site under fluoroscopy. They reported successful bone union in 53 of the 60 patients and found a correlation between the effectiveness of bone healing and the number of progenitor cells present in the aspirate.

 

The number and concentration of osteoprogenitor cells present in the bone marrow aspirates of the 7 patients who did not unite was significantly less than the patients in whom union was successfully achieved. All 7 patients who failed to heal were treated with a bone marrow graft that contained less than 1,000 progenitor cells/cm3 and less than 30,000 progenitor cells.

With the number of patients included in the study, the age, sex, and comorbidities of the patients did not significantly effect the success of treatment (P>.05).

Correct Answer: Number and concentration of stem cells implanted

 

 

3993. (3439) Q10-4364:

The number of stem cells obtained from an iliac crest bone marrow aspiration is influenced by:

 

1) Total volume of bone marrow obtained

3) Needle diameter used to aspirate bone marrow

2) Volume of each individual aspirate

5) Location selected for aspiration

4) Solution used to store the bone marrow

 

Muschler and colleagues found that the number of progenitor cells present in bone marrow aspirates was influenced by the volume of the aspirate. When a large volume was aspirated, the osteoprogenitor cells were diluted with peripheral blood. They recommended that the volume aspirated from any one site should not be greater than 2 cc.Correct Answer: Volume of each individual aspirate

 

 

3994. (3440) Q10-4365:

Which of the following statements describes the effect of parathyroid hormone (PTH) (1-34) on bone:

 

1) PTH (1-34) enhances bone formation when administered intermittently.

3) PTH (1-34) results in bone resorption when administered intermittently.

2) PTH (1-34) enhances bone formation when present continuously.

5) PTH (1-34) impairs osteoclast function when administered intermittently.

4) PTH (1-34) results in decreased osteoblast number and activity when administered intermittently.

 

While continuous exposure to PTH results in bone resorption, intermittent administration results in bone formation by increasing osteoblast number and activity. The catabolic effects of PTH are related to its activation of osteoclasts, likely through the direct activation of osteoblasts. While continuous exposure to PTH leads to increased osteoclastic activity, intermittent (once daily) exposure to PTH has been shown to preferentially stimulate osteoblast activity over osteoclast activity.Correct Answer: PTH (1-34) enhances bone formation when administered intermittently.

 

3995. (3469) Q10-4406:

Tricalcium phosphate is a bone graft substitute that can be best described as:

 

1) Osteogenic

3) Osteoinductive

2) Osteoconductive

5) None of the above

4) Both osteoconductive and osteoinductive

 

Tricalcium phosphate is an osteoconductive bone graft substitute. Although its hydrophilic nature may allow wicking of blood and bone marrow elements into the internal structure, it does not have any significant osteogenic properties. Osteogenesis refers to the property of autogenous bone grafts or bone marrow that provide live osteoblasts or stem cells that can differentiate into osteocytes. Osteoinduction refers to the process in which specific proteins, such as bone morphogenetic proteins, signal host stem cells to differentiate into bone cells rather than into other types of tissue.

 

Correct Answer: Osteoconductive

 

 

3996. (3487) Q10-4427:

Bone marrow-derived mesenchymal stem cells have the potential to differentiate into which tissue types:

 

1) Only bone

3) Bone, cartilage, tendon, muscle, and fat

2) Bone and blood cells

5) Bone, cartilage, tendon, muscle, and kidney

4) Bone, cartilage, muscle, and skin

 

Bone marrow-derived mesenchymal stem cells have the potential to differentiate into bone, cartilage, tendon, muscle, and fat.Correct Answer: Bone, cartilage, tendon, muscle, and fat

 

 

3997. (3490) Q10-4432:

In the treatment of postmenopausal osteoporosis, recombinant human parathyroid hormone teriparatide (Forteo, Eli Lilly) is administered:

 

1) Weekly, by injection, for less than 2 years

3) Daily, by injection, for less than 2 years

2) Weekly, by injection, for the patientâs entire life

5) Monthly, orally, for less than 2 years

4) Daily, by injection, for the patientâs entire life

 

Teriparatide is administered as a subcutaneous injection into the thigh or abdominal wall, once daily, at a recommended dosage of 20 µg. Because the safety and efficacy of teriparatide has not been evaluated beyond 2 years of treatment, its use for longer than 2 years is not recommended.

 

Ibandronate sodium (Boniva, Roche Laboratories), a nitrogen containing biphosphonate, is another class of medication that has been approved for the treatment of postmenopausal osteoporosis. It is available in both oral and injectable forms and may be administered monthly. The mechanism of action is programmed cell death of the osteoclast.

 

Correct Answer: Daily, by injection, for less than 2 years

 

3998. (3493) Q10-4435:

The mechanism by which intermittent parathyroid hormone (PTH) administration enhances fracture healing has been suggested to be all of the following except:

 

1) Stimulation of proliferation and differentiation of osteoprogenitor cells

3) Enhanced osteoclastogenesis during the callus-remodeling phase

2) Increased production of bone matrix protein

5) Accelerating bone mineralization during healing

4) Preferentially stimulating osteoclast activity over osteoblast activity

 

Nakajima and colleagues also studied the effect of daily PTH on fracture healing in a rat femur fracture model but at a dose of only 10 µg/kg. They found significantly increased bone mineral density and ultimate load to failure at days 28 and 42 after fracture. In addition, they investigated the underlying mechanism by analyzing the number of subperiosteal osteoprogenitor cells, the expression of type 1 collagen, osteonectin, alkaline phosphatase, and osteocalcin mRNA. Their results suggested that the enhanced early callus formation seen in PTH-treated animals was due to the stimulation of proliferation and differentiation of osteoprogenitor cells, increases in the production of bone matrix proteins, and enhanced osteoclastogenesis during the callus-remodeling phase.

 

While continuous exposure to PTH leads to increased osteoclastic activity, intermittent (once daily) exposure to PTH has been shown to preferentially stimulate osteoblast activity over osteoclast activity.

Correct Answer: Preferentially stimulating osteoclast activity over osteoblast activity

 

 

3999. (3511) Q10-4463:

Distinguishing characteristics of stem cells include that they are undifferentiated cells, can be induced to differentiate into the cell types of specific tissues, and:

 

1) Can induce differentiated cells to become stem cells

3) Can be obtained only from embryos

2) Are capable of self-replicating for indefinite periods in culture

5) Can only be induced to differentiate into specific cell types when in cell culture

4) Have only a single copy of DNA

 

Stem cells have three important distinguishing characteristics:

  1. Stem cells are undifferentiated cells.

  2. Stem cells are capable of self-replicating for indefinite periods in culture.

  3. Under certain physiologic or experimental conditions, stem cells can be induced to differentiate into the cell type of specific tissues, such as cartilage or bone.

 

Stem cells can be obtained from both embryos (embryonic stem cells) and from various adult tissues (adult stem cells). Correct Answer: Are capable of self-replicating for indefinite periods in culture

 

4000. (3670) Q10-7447:

Bioactive glasses are indicated for use in:

 

1) Subchondral fractures requiring a mechanically strong bone graft substitute

3) All open fractures

2) Open tibial fractures treated with an intramedullary nail

5) Dental and maxillofacial applications only

4) Bone defects that are not intrinsic to the stability of the bony structure

 

Bioactive glasses are indicated for use in the treatment of bone voids or gaps that are not intrinsic to the stability of the bony structure. These defects may be surgically created bony defects or defects resulting from traumatic bone injury. Bioactive glass products have been used as a bone graft substitute in dentistry since 1993 and were approved for orthopedic use in 2002.Correct Answer: Bone defects that are not intrinsic to the stability of the bony structure

 

4001. (3671) Q10-7448:

Bioactive glass bone graft substitute is described as osteostimulative. The term osteostimulation refers to the stimulation of what aspect of bone regeneration following implantation of bioactive glass:

 

1) Osteoblast proliferation and differentiation

3) Osteocyte proliferation and differentiation

2) Osteoclast proliferation and differentiation

5) Upregulation of bone morphogenetic protein

4) Mesenchymal stem cell proliferation and differentiation

 

The manufacturer of NovaBone describes bioactive glass as osteostimulative. They define osteostimulation as an active stimulation of osteoblast proliferation and differentiation. After implantation, surface reactions result in the absorption of the device material and concurrent new bone tissue formation. These surface reactions result in an osteostimulative effect.

 

In vitro studies comparing bioactive glass to other osteoconductive bone graft substitutes have shown increased levels of DNA synthesis and increased levels of the osteoblast markers osteocalcin and alkaline phosphatase.

Correct Answer: Osteoblast proliferation and differentiation

 

 

4002. (3672) Q10-7449:

The major component of bioactive glass is:

 

1) Calcium phosphate

3) Silica

2) Tricalcium phosphate

5) Sodium oxide

4) Calcium oxide

 

Available bioactive glass beads are composed of silica (45%), calcium oxide (24.5%), disodium oxide (24.5%), and pyrophosphate (6%).Correct Answer: Silica

 

 

4003. (3673) Q10-7450:

Resorption of bioactive glass is characterized as:

 

1) Extremely slow, with the majority of material remaining for decades

3) Fast, with the majority resorbed within 6 to 12 weeks

2) Slow, with the majority of material remaining for years

5) Occurring by cellular mechanisms only

4) Occurring by chemical dissolution only

 

Bioactive glass undergoes resorption by a combination of cellular mechanism and chemical dissolution. Upon implantation, the surface area of bioactive glass increases multifold. This provides greater exposure to interstitial fluid and blood resulting in faster resorption. The bioactive glass particles are gradually consumed as they are constantly re-exposed during bone remodeling.

Unlike hydroxyapatite, the majority of bioactive glass material is fully resorbed within 6 to 12 weeks after implantation.Correct Answer: Fast, with the majority resorbed within 6 to 12 weeks

 

 

4004. (3674) Q10-7451:

Bone formation that occurs following implantation of bioactive glass occurs in what location:

 

1) Only along the periphery of the grafted material

3) Throughout the grafted material

2) Only in the center of the grafted material

5) First in the center, gradually increasing toward the periphery over several months

4) First along the periphery of the material, with creeping substitution into the center taking several months

 

Bone formation in a critical-sized rabbit femoral defect model was greater and occurred faster with bioactive glass than with hydroxyapatite. At 2 weeks, the investigators observed visible bone throughout the bioactive glass material, whereas bone formation was only beginning to form along the margins of the hydroxyapatite.Correct Answer: Throughout the grafted material

 

4005. (3675) Q10-7452:

In this approval process, the manufacturer of a bone graft substitute product provides data showing that the new product is substantially equivalent to an already approved legally marketed product commercially distributed before the May 28, 1976, Medical Device Amendment.

 

1) Premarket application

3) Humanitarian device exemption

2) 510(k) premarket notification

5) Class II device application

4) Class I device application

 

The majority of bone graft substitute products on the market have been approved through the 510(k) process. In this process, the manufacturer must provide data showing that the new product is substantially equivalent to an already approved legally marketed device, which includes products commercially distributed before the May 28, 1976, Medical Device Amendment. This typically requires only laboratory and animal testing to establish safety and equivalency.Correct Answer: 510(k) premarket notification

 

 

4006. (3676) Q10-7453:

Requirements for a device approved through the humanitarian device exemption (HDE) process include which of the following:

 

1) The device is intended for a disease affecting fewer than 4000 individuals each year.

3) The device requires approval of the institutional review board where used.

2) The manufacturer is not permitted to make a profit from the device.

5) None of the above.

4) All of the above.

 

To make new drugs available to select patients, the FDA initiated a humanitarian device exemption (HDE) process in 1996. This process of drug and device approval is âintended to benefit patients by treating a disease that affects fewer than 4000 individuals in the US per year.â To obtain approval the manufacturer must show safety, but does not need to prove efficacy. Additionally, the HDE process requires that the drug or device cannot make a profit for the manufacturer and requires institutional review board approval at the institutions where it is used.Correct Answer: All of the above.

 

 

4007. (3677) Q10-7454:

The differences between a class I and class III device include:

 

1) Class I devices are generally more complex than class III devices.

3) Increased requirements are needed to show safety and efficacy for class III devices.

2) An increased level of risk of injury exists from a class I device.

5) Sufficient information exists to assure safety and effectiveness for class III devices solely through general or special controls.

4) Class III devices are subject to general controls, whereas class I devices are not.

 

A three-tiered classification was introduced by the Cooper Commission to stratify the risks of medical devices. Medical products classified as class I devices are subject to the least regulatory control. They present minimal potential harm to the user and are often simpler in design than class II or class III devices. Class I devices are subject to âgeneral controls,â which include registration and listing, prohibitions against adulteration and misbranding, notification, repair or replacement or refund, recall, records and reports, and adherence to good manufacturing practices.

 

Medical products classified as class II devices are those for which general controls alone are insufficient to assure safety and effectiveness, but for which existing methods are available to provide assurances of safety and efficacy. Class II medical devices are subject to general controls as described for class I devices, as well as also to special controls such as postmarket surveillance studies and performance standards.

 

Class III is the most stringent regulatory category for medical devices. Class III devices are those for which insufficient information exists to assure safety and effectiveness solely through general or special controls. Class III devices present a potential unreasonable risk of illness or injury as a result of their use. Premarket approval is the required process of scientific review to ensure the safety and efficacy of class III devices.

 

Correct Answer: Increased requirements are needed to show safety and efficacy for class III devices.

 

4008. (3678) Q10-7455:

An investigational device exemption (IDE):

 

1) Allows a new device to be used in a clinical study to collect safety and efficacy data required for a premarket application

3) Allows rapid approval of a class I device such as a manual surgical tool associated with a low risk of injury

2) Allows use of a device intended for a disease affecting fewer than 4000 individuals each year

5) Allows rapid approval of a device that is substantially equivalent to an already approved legally marketed device

4) Allows use of a class II device with âspecial controlsâ requiring postmarket surveillance studies

 

An IDE allows a new device to be used in a clinical study to collect the safety and efficacy data required for a premarket application. A humanitarian device exemption is the process of drug and device approval that is âintended to benefit patients by treating a disease that affects fewer than 4000 individuals in the US per year.â The process in which a manufacturer provides data showing that the new product is substantially equivalent to an already approved legally marketed device is the premarket notification process known as a 510(k).Correct Answer: Allows a new device to be used in a clinical study to collect safety and efficacy data required for a premarket application

 

 

4009. (3679) Q10-7456:

Approval of bone graft substitute products under a 510(k) process requires:

 

1) A large-scale pivotal human clinical study to prove efficacy

3) Typically only laboratory and animal testing data

2) A preliminary human clinical study to show safety

5) That the product be a human tissue that has only been minimally manipulated

4) No specific testing data because the product is similar to a product already being marketed

 

Many medical devices receive approval through a less stringent premarket notification process known as a 510(k). In this process, the manufacturer must provide data showing that the new product is substantially equivalent to an already approved legally marketed device, which includes products commercially distributed before the May 28, 1976, Medical Device Amendment. This typically requires only laboratory and animal testing to establish safety and equivalency.Correct Answer: Typically only laboratory and animal testing data

 

 

4010. (3680) Q10-7457:

Prostaglandin E2 has been shown to be a potent anabolic agent in bone. The anabolic effects on bone are mediated through which receptor(s):

 

1) EP1

3) EP3

2) EP2

5) EP2 and EP4

4) EP4

 

The action of PGE2 is mediated by its binding to four cell surface receptor subtypes: EP1, EP2, EP3, and EP4. Activation of EP2 and EP4 receptor subtypes causes an elevation of cellular cAMP levels. In bone, the anabolic action of PGE2 has been linked to elevated cAMP levels, suggesting that the EP2 and EP4 receptor subtypes play a role in bone formation. Bone anabolic activity is mediated by the EP2 and EP4 receptor subtypes, whereas the objectionable side effects are mediated by the EP1 and EP3 receptor subtypes.Correct Answer: EP2 and EP4

 

4011. (3681) Q10-7458:

In a phase 2 clinical study, the main focus is on what aspect of drug or device evaluation:

 

1) Safety alone

3) Effectiveness for a specific condition

2) Safety and tolerability

5) Postmarketing surveillance required by the US Food and Drug Administration (FDA)

4) Risk-benefit ratio

 

Phase 2 clinical studies focus on the effectiveness of the drug to treat the particular condition, whereas phase 1 clinical trials focus on safety and tolerability of the new medication. Phase 2 clinical trials typically involve several hundred patients who have the condition that the experimental medicine is intended to treat. Phase 3 trials test the results of earlier trials in larger populations and gather further information about the effectiveness and safety of the drug or device. This phase usually involves several hundred to several thousand patients enrolled in one or more pivotal clinical trials. Phase 3 trials generally provide the basis for the risk-benefit assessment of a new drug. Phase 4 trials are also called postmarketing studies and are conducted after the drug has been approved by the FDA. These trials allow researchers to collect information about the long-term risks and benefits of the product.Correct Answer: Effectiveness for a specific condition

 

 

 

4012. (3682) Q10-7459:

The usefulness of prostaglandin E2 to stimulate bone repair following fracture is limited by its:

 

1) High cost

3) Lack of anabolic effect on bone repair

2) Side effects that include diarrhea, lethargy, and flushing

5) Inability to maintain local concentration after injection

4) Long-term complications, including development of osteosarcoma

 

The use of prostaglandins in human clinical studies has been complicated by multiple and severe adverse reactions. Side effects of the use of PGE2 include diarrhea, lethargy, and flushing. PGE2 has been shown to have anabolic effects on bone regeneration.

Long-term complications have not been reported. The side effects cease following discontinuation of use.

Researchers, hoping to harness the anabolic effect of PGE2 while avoiding its unpleasant side effects, sought to identify EP2 and EP4 receptor-selective agonists.Correct Answer: Side effects that include diarrhea, lethargy, and flushing

 

 

4013. (3683) Q10-7460:

Preclinical results of the effects of the EP2 receptor-agonist named CP-533,536 have shown:

 

1) Ability to form bone only in non_critical-sized defect models

3) Anabolic effects only when administered systemically

2) Dose-dependent formation of bone in a critical-sized defect

5) Effectiveness only in lower order animals (rats)

4) Significant bone resorption (osteopenia) following cessation of treatment

 

Preclinical results of the effects of the EP2 receptor agonist named CP-533,536 have been reported in two different publications.

In one canine study, a critical-sized (1.5-cm) defect was created in the canine ulna and a collagen sponge placed in the defect. Animals received injections of CP-533,536 into the defect site for 3, 7, or 14 days. Radiographic evidence of bony bridging of the critical size defect was seen in 20%, 30%, and 60% of the cases at 10 weeks.

 

In a second canine study, the same critical-sized defect was created and filled with a PLGH matrix (ATRIGEL delivery system, Atrix Laboratories, Fort Collins, CO). Animals were randomized to receive either 1.0 mL of the matrix alone, 50 mg of CP-533,536 in

1.0 mL of matrix, 10 mg of CP-533,536 in 1.0 mL of matrix, or 10 mg of CP-533,536 in 0.2 mL of matrix. No additional injections were given during the study, with the matrix offering a gradual release of the compound locally. None of the animals treated with the matrix alone healed. In animals treated with 50 mg of CP-533,536, half of the animals showed radiographic bridging of new bone at 24 weeks. Even greater bone healing was seen in the two groups treated with the lower dose of CP-533,536, with 75% of animals showing radiographic bridging at 24 weeks.

 

Correct Answer: Dose-dependent formation of bone in a critical-sized defect

 

4014. (3684) Q10-7461:

Which of the following agents currently in clinical trials offers potential for use in bone repair following fracture:

 

1) Prostaglandin E1

3) Prostaglandin E2 combined with an anticholinergic agent to minimize side effects

2) Prostaglandin E2

5) Prostaglandin E2 combined with an EP2 receptor-selective agonist

4) PGE2 EP2 receptor-selective agonist

 

The use of prostaglandins in human clinical studies has been complicated by multiple and severe adverse reactions. Side effects of the use of PGE2 include diarrhea, lethargy, and flushing.

Researchers, hoping to harness the anabolic effect of PGE2 while avoiding its unpleasant side effects, sought to identify EP2 and EP4 receptor-selective agonists. Researchers at Pfizer Global Research and Development have reported on a highly selective and potent EP2 receptor agonist named CP-533,536. CP-533,536 is currently in phase 2 clinical trials for bone healing.

Correct Answer: PGE2 EP2 receptor-selective agonist

 

 

4015. (3685) Q10-7462:

Platelet-derived growth factor (PDGF) is currently approved in the United States for which of the following indications:

 

1) Periodontal bone defects

3) Periodontal bone defects and diabetic foot wounds

2) Diabetic lower extremity wounds

5) Open tibial shaft fractures treated with an intramedullary nail

4) As an alternative to autogenous graft for ankle fusion

 

rhPDGF-BB is currently approved in the United States for use in periodontal bone defects and healing diabetic lower extremity soft tissue wounds due to neuropathy.

Preliminary results of PDGF as an alternative to autogenous bone graft in hindfoot and ankle fusions are promising, and a larger pivotal trial is underway. However, at this time (2008) the product has not been approved for this indication.

Correct Answer: Periodontal bone defects and diabetic foot wounds

 

 

4016. (3686) Q10-7463:

During the initial phase of the wound healing process, platelet-derived growth factor (PDGF) acts to attract and activate which cells:

 

1) Osteoprogenitor cells

3) Bone morphogenetic protein (BMP)-producing cells

2) Neutrophils and macrophages

5) Platelets

4) Vascular endothelial growth factor (VEGF)-producing cells

 

Following injury, bone repair is initiated by formation of a blood clot and activation of the coagulation cascade. Platelets accumulate at the fracture site and release their cytokine-laden granules, including PDGF-AB, PDGF-AA, PDGF-BB, and PDGF-CC. These PDGFs initially attract and activate neutrophils and macrophages. These neutrophils and macrophages then provide an ongoing supply of PDGF and other growth factors that play an important role in granulation tissue formation. Later in the wound healing process, PDGFs also exert mitogenic and chemoactactic effects on a variety of mesenhcymal derived cells, including fibroblasts, osteoblasts, and chondrocytes.Correct Answer: Neutrophils and macrophages

 

 

4017. (3687) Q10-7464:

This platelet-derived growth factor (PDGF) is considered the universal isoform because it binds to all known PDGF receptor isotypes. This is the isoform which is being used and studied clinically:

 

1) rhPDGF-AA

3) rhPDGF-CC

2) rhPDGF-BB

5) rhPDGF-AB

4) rhPDGF-DD

 

PDGF is a family of proteins that includes five forms: four homodimeric proteins, PDGF-AA, PDGF-BB, PDGF-CC, and PDGF-DD; and one heterodimeric protein, PDGF-AB. PDGF-BB is considered the universal isoform because of its ability to bind to all known PDGF receptor types.Correct Answer: rhPDGF-BB

 

 

4018. (3688) Q10-7465:

Local application of platelet-derived growth factor (PDGF) in a collagen carrier to a rabbit tibial osteotomy site resulted in:

 

1) Decreased callus density and volume

3) Full restoration of mechanical strength (similar to intact bone) 4 weeks after treatment

2) No effect due to the short half-life of PDGF

5) A high rate of wound healing complications

4) Formation of new bone only on the periosteal surface

 

The effect of exogenous PDGF was studied in a rabbit tibial osteotomy model. Each osteotomy was injected with collagen or collagen containing 80 μg of rhPDGF-BB. Radiographs at 2 and 4 weeks showed an increase in callus density and volume around the PDGF-treated osteotomies. Mechanical testing of the osteotomized and intact tibia was performed at 28 days. The strength of the osteotomy site recovered to the level of the intact bone in animals treated with PDGF, while the control group was statistically weaker than the nonoperated (contralateral) tibia. Histologically, the PDGF-treated tibiae displayed a more robust and advanced state of osteogenic differentiation, both endosteally and periosteally, than the control osteotomies.Correct Answer: Full restoration of mechanical strength (similar to intact bone) 4 weeks after treatment

 

 

 

4019. (3689) Q10-7466:

Preliminary results of a clinical study of platelet-derived growth factor (PDGF) as an alternative to autogenous bone graft in ankle and hindfoot fusions shows that PDGF:

 

1) Must be applied daily to the site to achieve a positive effect

3) Has a higher rate of wound complication

2) Must be administered systemically by daily injection to achieve a positive effect

5) Has a similar rate of bony fusion, documented by computed tomography (CT) scan

4) Forms heterotopic bone

 

The results of pilot study that compared the effectiveness of rhPDGF-BB in a beta-tricalcium phosphate matrix with autogenous bone graft in hindfoot and ankle fusions were presented in 2007. Investigators found equivalent osseous bridging on CT scans at 6 and 12 weeks after surgery, and a > 85% fusion rate for both groups at 24 weeks.

 

Positive results of this study have lead to a larger pivotal randomized trial that is expected to enroll 396 patients. As of August 8, 2008, 220 patients had been enrolled in this randomized controlled study. Additional presentations on the use of rhPDGF as an alternative to autogenous bone graft have also reported positive results.

 

Correct Answer: Has a similar rate of bony fusion, documented by computed tomography (CT) scan

 

 

4020. (3690) Q10-7467:

Compared with other recombinant human proteins that are used clinically, recombinant human bone morphogenetic proteins (rhBMPs) are priced:

 

1) Far lower than other recombinant proteins

3) Higher than most other recombinant proteins

2) Similar to other recombinant proteins

5) There is no comparison, because rhBMP is the only recombinant human protein in clinical use.

4) Significantly higher than other recombinant proteins

 

The cost of rhBMP is in line with the costs for other commercially available recombinant proteins. For example, the treatment costs for recombinant human erythropoietin is between $6000 and $10,000.Correct Answer: Similar to other recombinant proteins

 

 

4021. (4064) Q10-7468:

A thorough economic analysis to evaluate the cost effectiveness of a particular treatment should include which of the following factors:

 

1) Cost of the product alone

3) Direct and indirect costs

2) Direct costs alone

5) Cost of the product, direct costs, and intangible costs

4) Direct, indirect, and intangible costs

 

A thorough economic analysis of an individual product or treatment should include direct costs, indirect costs, and intangible costs. Cost of the product is just one portion of the direct costs. Examples of each of these types of costs are provided in the table below:

 

Table 2. Type of economic costs associated with healthcare delivery

Direct costs

Indirect Costs

Intangible Costs

Medical

Nonmedical

 

 

Surgical interventions

Transportation of patients and families during treatment

Lost productivity (lost wages)

Quality of life (associated with pain, suffering, and grief)

Personnel costs

Lodging of patients and families during treatment

Rehabilitation and skilled nursing care

Quality-adjusted life years

Supplies costs

 

Disability payments

Psychosocial parameters

Hospitalization costs

 

Legal costs

 

Diagnostic tests

 

Insurance administration costs

 

Medications

 

 

 

Outpatient visits

 

 

 

Physical therapy

 

 

 

 

Correct Answer: Direct, indirect, and intangible costs

 

4022. (3691) Q10-7469:

The high cost of recombinant human proteins is primarily due to:

 

1) The addition of an extremely high profit margin for the manufacturer

3) The high cost of marketing required to convince surgeons to use such an experimental product

2) The addition of an extremely high payment to the patent holder for each individual use

5) The high cost of the carrier with which the product is delivered

4) The high costs required to produce the product

 

Whereas all of these factors must be considered in setting the price of a recombinant product, the high cost required to produce a recombinant protein is the primary reason for the productâs cost. The cost to design, build, validate, and qualify an industrial size bioreactor for the production of a recombinant protein costs from $400 million to $2 billion. This process can take 3 to 5 years to complete.

 

In addition to the initial set-up investment, substantial costs are incurred during the production of a recombinant human protein. These expenses include the costs required for equipment sterilization, cell culture production, recovery, purification, viral filtration, in-process testing, and aseptic packaging. The production of a single batch of recombinant protein can take up to 100 highly trained personnel working in continuous shifts for more than a month. Continuous monitoring is required throughout the production process, with more than 200 analytic tests performed during the production run.

 

Product pricing also needs to account for the costs of the carrier, costs of distribution and marketing, costs for royalties to the inventor/patent holder, costs for ongoing required research, and finally costs to provide a profit to the commercial manufacturer.Correct Answer: The high costs required to produce the product

 

 

4023. (3692) Q10-7470:

Economic analysis of the clinical use of recombinant human bone morphogenetic protein (rhBMP) has shown:

 

1) Cost effectiveness only for spinal surgery

3) Cost effectiveness only for tibial nonunions

2) Cost effectiveness only for open tibial fractures

5) No evidence of cost effectiveness for any indication

4) Cost effectiveness for spinal surgery, open tibial fractures, and tibial nonunions

 

Even though there is great variation in economic analysis methods and underlying assumptions, several published reports have suggested that the use of rhBMP is cost effective. These studies include spine surgery, open tibial fractures, and tibial nonunions. Conflicting results may be obtained depending on whether the analysis is done from the viewpoint of the hospital (which may not receive additional compensation for the product use), from the viewpoint of the payor, or from the viewpoint of society as a whole.

 

All health care costs over a 3-month period were analyzed in a prospective study of 102 patients undergoing lumbar spine fusion randomized to receive rhBMP-2 or iliac crest bone graft. Total payor expenditures for the 3-month perioperative period averaged

$33,860 in the rhBMP-2 group compared with $37,337 in the iliac crest bone graft group. Patients treated with rhBMP-2 had a shorter mean operative time and a shorter hospital length of stay. Patients treated with iliac crest bone graft had more frequent and more severe complications, leading to increased costs. Also, a higher rate of inpatient rehabilitation was seen in the iliac crest bone graft group.

 

Using economic modeling and data from published level I and level II studies, Ziran and colleagues compared the costs of rhBMP use in the management of open tibial fractures. They evaluated the costs of initial acute fracture treatment and the costs for subsequent treatment of nonunion or deep infection. They compared the total societal costs for three different scenarios. In the first scenario, rhBMP was not used during any of the treatments. In the second scenario, rhBMP was used both acutely and during any subsequent procedures for the treatment of nonunion or deep infection. In the third scenario, rhBMP was not used during the acute treatment but was only used during the treatment of nonunion or deep infection. In all three scenarios, the investigators reported little difference in total societal costs. They calculated the aggregate cost of treatment in the first scenario (no use of rhBMP) as $572 million per year. In the second scenario (use of rhBMP at all procedures), the aggregate cost of treatment was reduced slightly to $568 million per year. In the third scenario (use of rhBMP only for secondary nonunion or infection treatment), the aggregate cost of treatment was slightly greater at $575 million per year.

 

The costs incurred to treat 25 nonunions with rhBMP-7 were examined and compared with prior treatment of those patients. The investigators found that the overall costs of treatment of the procedures in which rhBMP-7 was used were 47% less than the costs of the prior treatment in which rhBMP-7 had not been used. The mean hospital stay and costs of treatment before receiving

rhBMP-7 was calculated at 26.8 days and £13,844, compared with 7.8 days and £7338 for the procedures in which rhBMP-7 was used.Correct Answer: Cost effectiveness for spinal surgery, open tibial fractures, and tibial nonunions

 

4024. (3693) Q10-7471:

Commercial production of a recombinant human protein requires:

 

1) Limited capital investment

3) No specific testing during production

2) Only basic laboratory equipment and a few personnel

5) Continuous monitoring and numerous analytic tests throughout the production process

4) Testing only of the final product to ensure sterility and efficacy

 

Extensive capital investment is required to produce a recombinant protein for commercial use. The cost to design, build, validate, and qualify an industrial size bioreactor for the production of a recombinant protein costs on average from $400 million to $2 billion. This process can take from 3 to 5 years to complete.

 

The production of a single batch of recombinant protein can take up to 100 highly trained personnel working in continuous shifts for more than a month. Continuous monitoring is required throughout the production process, with more than 200 analytic tests performed during the production run.Correct Answer: Continuous monitoring and numerous analytic tests throughout the production process

 

 

4025. (3943) Q10-8181:

Which of the following is considered the gold standard of bone graft material:

 

1) Ceramics

3) Autograft

2) Bone marrow aspirates

5) Bone morphogenetic proteins

4) Blood product isolates

 

Autograft is still the âgold standardâ bone graft material. Autograft is generally harvested from the iliac crest and provides osteogenic, osteoinductive, and osteoconductive factors. However, autograft is associated with significant morbidity in up to one-third of patients. Additionally, despite the use of autograft, pseudarthrosis remains a concern. For these reasons, materials are being investigated for use as adjuncts or alternatives to this grafting material.Correct Answer: Autograft

 

 

4026. (4019) Q10-8272:

Which of the following is NOT a mechanism of action of physiologic Hyaluronan?

 

1) Large molecule exclusion properties

3) Desensitization of nociceptive fibers

2) Interaction with cellular signaling such as CD44

5) Decrease in synovial fluid viscosity

4) Alteration of cell adhesion molecules

 

Hyaluronan has several traits that are thought to contribute to its mechanism of action. It modulates synovial cell and chondrocyte activity, nociceptive pain fibers, cell adhesion, and has large molecule exclusion properties. Hyaluronan also increases viscosity of the synovial fluid, which produces a mechanical benefit for sheer and compressive force distribution.Correct Answer: Decrease in synovial fluid viscosity

 

 

4027. (4020) Q10-8273:

Which of the following is NOT thought to be a maladaptive response of chondrocytes and cartilage to loading in osteoarthritis?

 

1) Increased production of aggrecanases

3) Shifting of the metabolic balance towards catabolism

2) Shifts in water and proteoglycan content

5) Increased production of matrix metalloproteinases

4) Subchondral sclerosis

 

In osteoarthritis, chondrocytes subjected to loading shift their metabolism towards catabolic functions, including production of aggrecanases and metalloproteinases. There is also a shift in water and proteoglycan content of cartilage. Subchondral sclerosis is a well-described feature of osteoarthritis, but is thought to be related to changes in bone metabolism.Correct Answer: Subchondral sclerosis

 

4028. (4016) Q11-8269:

A 45-year-old heavy laborer presents with 2 years history of pain over the medial aspect his right knee. On examination, the knee has an alignment of 5° varus with a range of motion from 0-105°. He weighs 185 lbs., has no evidence of ligamentous instability and has failed all conservative management. This patient is not a suitable candidate for unicondylar knee replacement because of:

 

1) Age

3) Knee alignment

2) Occupation

5) Limited preoperative knee flexion

4) Weight

 

When qualifying a patient for arthroplasty, one must consider the patient's anatomy and his/her goals following surgery. In this case, the patient has all but 1 of the minimum requirements for medial unicondylar knee arthroplasty. Because this patient is a heavy laborer, an unloading osteotomy of the knee may be a more suitable therapeutic choice in order to allow the patient to return to his prior activities following surgery.

 

Correct Answer:

Occupation

 

 

 

4029. (4017) Q11-8270:

A 67-year-old woman presents with 5 years history of progressive right knee pain. She complains of activity related knee pain over the anterior aspect of the knee. Her past medical history has included history of breast cancer (in remission), 3 prior right knee arthroscopies, and multiple series of viscosupplementation injections to her right knee. She walks with a cane and a slight lurch. Examination of the knee reveals varus alignment of 3°, range of motion from 0° to 115°, and no evidence of instability. Radiographs of her knee reveal moderate narrowing of the medial aspect of the knee joint. The next most appropriate step in her clinical management is:

 

1) Magnetic resonance imaging of the lumbar spine

3) Consult pain management

2) Bone scan to rule out metastatic disease

5) Radiographs of her pelvis

4) Total knee or unicondylar knee arthroplasty

 

This woman has presented with a long history of unremitting right knee pain for which she has undergone multiple surgical procedures and injections. While she does have some clinical evidence of knee arthrosis, the severity of her symptoms does not correlate with the severity of her joint disease (radiographs). She walks with a lurch, which may suggest hip disease, a common source of referred pain to the knee. When evaluating a patient, one must take into account all potential causes of pain including adjacent joints.

 

Correct Answer: Radiographs of her pelvis

 

4030. (4018) Q11-8271:

A 62-year-old woman presents to the office with severe tricompartmental arthritis of both her knees. She is morbidly obese (body mass index 50) and is unable to walk or exercise because of her pain. Her left knee is more symptomatic, but she would like to have simultaneous bilateral total knee arthroplasty (TKA). The most appropriate next step in this patient's management is:

 

1) Left TKA to allow the patient to mobilize and lose weight followed by right TKA at least 3 months apart

3) Referral to a weight loss program including consultation with a bariatric surgeon

2) Simultaneous bilateral knee replacements

5) Viscosupplementation injections

4) Physical therapy

 

Studies show that super-obese patients (body mass index 50 or greater) have significantly increased complications when undergoing primary unilateral knee replacements. Consequently, simultaneous bilateral TKA would not be a reasonable choice for this patient. Others have also shown that weight following hip and knee replacements do not significantly change unless accompanied by lifestyle changes. In this situation, the safest course for this patient is to postpone surgery and pursue a preoperative weight loss program.

 

Correct Answer:

Referral to a weight loss program including consultation with a bariatric surgeon

 

 

 

4031. (4034) Q11-8292:

Which of the following is NOT a characteristic radiographic finding in osteoarthritis?

 

1) A. Subchondral cyst

3) C. Osteophyte formation

2) B. Osteopenia

5) E. Subchondral sclerosis

4) D. Joint space narrowing

 

The characteristic radiographic findings in osteoarthritis, which are constant across all joints, include joint space narrowing (often asymmetric), subchondral sclerosis, subchondral cysts, osteophytes and normal bone mineralization. These findings are demonstrated in varying levels of severity depending on the degree of osteoarthritis, and certain joints demonstrate certain findings more frequently. Osteopenia is not a common criterion for osteoarthritis as it is more commonly seen in rheumatoid arthritis. As patients age, there can certainly be both osteopenia and osteoarthritis present, but not necessarily related to each other as in rheumatoid arthritis.Correct Answer: B. Osteopenia

 

 

4032. (4035) Q11-8293:

Which of the following is NOT a characteristic radiographic finding in osteoarthritis?

 

1) Subchondral cyst

3) Osteophyte formation

2) Osteopenia

5) Subchondral sclerosis

4) Joint space narrowing

 

The characteristic radiographic findings in osteoarthritis, which are constant across all joints, include joint space narrowing (often asymmetric), subchondral sclerosis, subchondral cysts, osteophytes and normal bone mineralization. These findings are demonstrated in varying levels of severity depending on the degree of osteoarthritis, and certain joints demonstrate certain findings more frequently. Osteopenia is not a common criterion for osteoarthritis as it is more commonly seen in rheumatoid arthritis. As patients age, there can certainly be both osteopenia and osteoarthritis present, but not necessarily related to each other as in rheumatoid arthritis.Correct Answer: Osteopenia

 

4033. (232) Q12-342:

Which of the following is primarily responsible for lateral epicondylitis (tennis elbow):

 

1) Degeneration of the origin of the extensor carpi radialis longus

3) Degeneration of the origin of the extensor carpi ulnaris

2) Degeneration of the origin of the extensor carpi radialis brevis

5) Entrapment of the posterior interosseous nerve

4) Periostitis of the lateral epicondyle

 

The primary lesion of lateral epicondylitis is fibroangioblastic invasion of the origin of the extensor carpi radialis brevis. Tennis elbow usually responds to nonoperative measures, such as ice, anti-inflammatory medications, and activity modification.

Occasionally, steroid injections or surgical debridement becomes necessary.Correct Answer: Degeneration of the origin of the extensor carpi radialis brevis

 

 

 

4034. (669) Q12-922:

A 42-year-old recreational tennis player has had symptoms of lateral epicondylitis in his dominant elbow for 1 week. Appropriate management should involve what regimen of treatment:

 

1) Splinting elbow in flexion for 2 weeks

3) Injection of steroid and return to activity

2) Surgical release of the extensor origin at the lateral epicondyle

5) Nonsteroidal anti-inflammatory drugs, ice, and activity modification

4) Heat modalities and wrist extension exercises

 

With only a 1-week history of lateral epicondylitis of the elbow, the least invasive modalities of treatment must be employed. Nonsteroidal anti-inflammatory drugs, ice, and activity modification will usually improve the symptoms. If the symptoms continue to persist after 6 weeks of treatment, then a more invasive modality is necessary.Correct Answer: Nonsteroidal anti-inflammatory drugs, ice, and activity modification

 

 

4035. (670) Q12-923:

Following an extensive medial release for resistant medial epicondylitis, the most serious iatrogenic complication is:

 

1) Tardy ulnar nerve palsy

3) Medial elbow instability

2) Weak forearm pronation

5) Posterior interosseous nerve palsy

4) Weak thumb apposition

 

Medial elbow instability is the most serious complication of an extensive medial release for resistant medial epicondylitis.Correct Answer: Medial elbow instability

 

 

4036. (214) Q13-322:

Dislocation following total hip arthroplasty has been associated with all of the following except:

 

1) Posterior surgical approach

3) Use of a 22 mm prosthetic femoral head

2) Rheumatoid arthritis

5) Osteonecrosis

4) Impingement of the prosthetic neck on the acetabular component

 

Dislocation occurs following total hip arthroplasty approximately 1% to 10% of the time. The posterior approach has been associated with a higher dislocation rate. Rheumatoid arthritis, regardless of surgical approach employed, is also a risk factor. Use of a 22 mm femoral head can result in impingement of the prosthetic neck on the acetabular component that can result in dislocation. Patients with osteonecrosis have not been shown to have a higher dislocation rate following total hip arthroplasty.Correct Answer: Osteonecrosis

 

 

4037. (215) Q13-323:

Polyethylene wear debris after total hip arthroplasty is most commonly the result of:

 

1) Foreign body reaction

3) Wear at the articulation of the femoral head with the polyethylene cup

2) Third body wear

5) Use of a 32-mm prosthetic femoral head

4) Wear at the interface of the metal acetabular shell and the polyethylene liner

 

Polyethylene wear debris most often occurs at the articulation of the prosthetic femoral head with the polyethylene acetabular cup.Correct Answer: Wear at the articulation of the femoral head with the polyethylene cup

 

 

 

4038. (216) Q13-324:

Following cemented total hip arthroplasty, polyethylene wear rates average:

 

1) 0.05 mm per year

3) 0.2 mm per year

2) 0.1 mm per year

5) 0.4 mm per year

4) 0.3 mm per year

 

Acetabular wear rate following cemented total hip arthroplasty can be expected to be 0.1 mm per year.Correct Answer: 0.1 mm per year

 

 

 

4039. (217) Q13-325:

Of the following, the main difference between cemented and cementless femoral components in total hip arthroplasty for osteoarthritis is:

 

1) Higher incidence of thigh pain in cementless components

3) Higher dislocation rate in cementless components

2) Higher infection rate in cemented components

5) Higher incidence of abductor insufficiency in cemented components

4) Higher incidence of groin pain in cementless components

 

Thigh pain following prolonged activity can occur at the tip of a well-fixed cementless femoral component. This pain is usually not severe and responds to simple measures, such as over-the-counter analgesics and activity modification. Groin pain usually relates to problems with the acetabular component. Rates of infection, dislocation, and abductor insufficiency are similar between the two groups.Correct Answer: Higher incidence of thigh pain in cementless components

 

 

 

4040. (218) Q13-326:

Total hip arthroplasty in patients with ankylosing spondylitis most often results in which of the following:

 

1) Aseptic loosening

3) Dislocation

2) Infection

5) Groin pain

4) Heterotopic ossification

 

Up to 54% of patients with ankylosing spondylitis develop heterotopic ossification following total hip arthroplasty, although the majority of these will be asymptomatic. Prophylaxis in the form of single-dose radiation within 72 hours of surgery should be considered in this patient population.Correct Answer: Heterotopic ossification

 

 

4041. (219) Q13-327:

Prior to performing total hip arthroplasty in an otherwise healthy patient with rheumatoid arthritis and pain isolated to the hip, the following tests should be performed:

 

1) Knee radiographs

3) Serum C-reactive protein

2) Echocardiogram

5) Flexion and extension cervical spine radiographs

4) Liver function tests

 

Patients with rheumatoid arthritis are prone to develop cervical instabilities, especially at the C-1 to C-2 level. Up to 50% of rheumatoid patients with cervical instability may be asymptomatic and have a normal physical examination. Radiographic evaluation of the cervical spine is recommended prior to any operative procedure in this patient population to help avoid damaging the cervical spinal cord during endotracheal intubation.Correct Answer: Flexion and extension cervical spine radiographs

 

 

 

4042. (220) Q13-328:

The most important factor in controlling infection following total hip arthroplasty is the use of:

 

1) Perioperative antibiotics

3) Individual surgical exhaust systems

2) Laminar flow operating rooms

5) Limited verbal communication during the procedure

4) Ultraviolet operating room lighting

 

While all of the responses have been associated with lower infection rates, the use of perioperative antibiotics has the greatest impact on minimizing infection.Correct Answer: Perioperative antibiotics

 

 

 

4043. (221) Q13-329:

Immediately following total hip arthroplasty, a patient is noted to have a femoral nerve palsy of the operated extremity. Appropriate management should include:

 

1) Observation

3) Immediate electromyogram (EMG)

2) Immediate operative exploration

5) EMG at 2 weeks postoperative if no return of nerve function

4) Delayed operative exploration and nerve grafting in 2 weeks

 

Nerve injury following total hip arthroplasty is rare. Sciatic nerve injury, the most common, usually results from traction associated with lengthening of the operated extremity. Femoral nerve injury most commonly results from excessive retraction of the psoas muscle. Fortunately, the majority of these palsies resolve spontaneously.Correct Answer: Observation

 

 

 

4044. (226) Q13-335:

When performing total hip arthroplasty, the femoral component should be positioned in:

 

1) 10° to 20° retroversion

3) Neutral version

2) 5° to 10° retroversion

5) 10° to 20° anteversion

4) 5° to 10° anteversion

 

The femoral component should be positioned in 5° to 10° of anteversion during total hip arthroplasty.Correct Answer: 5° to 10° anteversion

 

 

4045. (227) Q13-336:

When performing total hip arthroplasty, the acetabular component should be positioned in:

 

1) 10° anteversion and 45° vertical inclination

3) Neutral version and 60° vertical inclination

2) 10° anteversion and 60° vertical inclination

5) 10° retroversion and 60° vertical inclination

4) 10° retroversion and 45° vertical inclination

 

The acetabular component should be placed in 10° anteversion and 45° vertical inclination during total hip arthroplasty.Correct Answer: 10° anteversion and 45° vertical inclination

 

 

 

4046. (4025) Q13-8278:

The most common cause of dislocation after primary total hip arthroplasty is

 

1) Implant failure

3) Component malposition

2) Infection

5) Neurologic dysfunction

4) Muscle weakness

 

Although neurologic dysfunction, soft tissue laxity, and loosening due to implant failure or infection contribute to THA instability, component mal-position is the leading cause of dislocation.Correct Answer: Component malposition

 

 

4047. (4026) Q13-8279:

Which of the following is NOT a consequence of acetabular shell malposition:

 

1) Fibrous ingrowth

3) Increased bearing wear

2) Increased fretting wear

5) Limited range of motion

4) Impingement

 

Mal-position leads to limited range of motion, impingement, and increased bearing and fretting wear. Fibrous ingrowth is most commonly a consequence of inadequate fixation, and excessive micro-motion.Correct Answer: Fibrous ingrowth

 

 

4048. (4027) Q13-8280:

Excessive anteversion of the acetabular cup may lead to :

 

1) Cup medialization

3) Leg length discrepancy

2) Posterior implant impingement

5) Dislocation with excessive internal rotation

4) Premature osteolysis

 

Excessive anteversion leads to anterior dislocation due to posterior component impingement. This most commonly occurs through extension and external rotation of the lower extremity. Excessive anteversion has little or no direct effect on medialization of the cup, leg length disparity, or premature osteolysis.Correct Answer: Posterior implant impingement

 

4049. (4028) Q13-8281:

Mechanical guide inaccuracy in cup placement during THA occurs due to:

 

1) Anatomic soft tissue variance

3) Poor implant fixation

2) Displaced fracture of acetabulum

5) Pelvic positional instability

4) Excessive motion between guide and implant

 

With adequate exposure, compensation for soft tissue variance is accomplished. Fracture is uncommon, as is gross motion between implant and bone. Provided the guide is used correctly, there is no appreciable motion between it and the implant. Changes in pelvic and patient position, however, will render the mechanical guide inaccurate.Correct Answer: Pelvic positional instability

 

 

4050. (4029) Q13-8282:

Excessive abduction of the acetabular shell may result in all of the following EXCEPT:

 

1) Edge loading

3) Osteolysis

2) Superior instability

5) Linear polyethylene wear

4) Superior cup migration

 

Edge loading, superior dislocation or subluxation, linear polyethylene wear and resultant premature osteolysis may all result from an excessively abducted cup. Superior cup migration is most commonly a consequence of a cup with very low abduction.Correct Answer: Superior cup migration

 

 

4051. (4030) Q13-8283:

Bone remodeling and adaptation is driven by all of the following EXCEPT:

 

1) Non-weightbearing

3) Skeletal loading in excess of physiologic amounts

2) Infection

5) Consistent short habitual loading

4) Local steroid injection

 

Bone cells are felt to respond to unique loading, short bursts of habitual loading and avoidance of non-weight bearing. Infection does not stimulate normal bone remodeling, nor does local steroid injection.Correct Answer: Consistent short habitual loading

 

 

4052. (4031) Q13-8284:

Which if the following is involved with local signaling in bone remodeling?

 

1) Hemoglobin A1C

3) Leukocyte esterase

2) Nitric oxide

5) Conjugated bilirubin

4) Alkaline phosphatase

 

Of those listed, alkaline phosphatase is most directly linked to local factors influencing bone remodeling.Correct Answer: Alkaline phosphatase

 

4053. (4032) Q13-8285:

Which of the following variables MOST strongly increases the stiffness of a cylindrical femoral stem?

 

1) Forged stem

3) Increasing stem length

2) Increasing stem radius

5) Physical therapy

  1. Slot in distal stem

     

    Of those variables listed, only an increase in stem radius would significantly increase stem cross-sectional area and hence, stiffness. Increasing length and a slot in the distal tip would decrease stiffness; no significant changes would occur as a result of cast or forging in the stem manufacturing.Correct Answer: Increasing stem radius

     

     

    4054. (4033) Q13-8286:

    The amount of bone mineral density loss that is necessary to become radiographically apparent is:

     

    1) 10%

    3) 30%

    2) 20%

  2. 50%

4) 40%

 

Studies have shown that a thirty percent loss in bone mineral density is necessary to consistently become visible on plain film lower extremity radiographs.Correct Answer: 30%

 

 

4055. (222) Q14-330:

Which of the following patients would most likely derive the greatest benefit from arthroscopic debridement of the knee:

 

  1. A 65-year-old woman with rheumatoid arthritis

3) A 65-year-old man with generalized knee pain and no mechanical symptoms

2) A 55-year-old man with a varus deformity and an osteoarticular lesion on the weight-bearing surface of the medial femoral condyle

5) A 50-year-old woman with insidious onset of medial knee pain without mechanical symptoms

4) A 55-year-old man with medial knee pain and symptoms of locking

 

Arthroscopic debridement in the treatment of degenerative conditions of the knee is most successful when symptoms are acute in onset, isolated to one area of the knee, and mechanical in nature. If the mechanical axis of the knee passes through an osteoarticular lesion, little benefit can be expected from arthroscopy. Arthroscopic treatment of the degenerative knee consists of removal of any torn meniscal fragments, loose bodies, and unstable chondral flap tears.Correct Answer: A 55-year-old man with medial knee pain and symptoms of locking

 

 

 

4056. (223) Q14-331:

Which of the following patients is the best candidate for unicompartmental arthroplasty of the knee:

 

1) A 60-year-old, 100 kg man with medial knee arthritis and a 10° fixed varus deformity

3) A 50-year-old, 80 kg manual laborer with medial knee arthritis

2) A 70-year-old, 70 kg man with medial knee arthritis and a 130° arc of motion

5) A 60-year-old, 50 kg woman with lateral knee arthritis and a 10° fixed valgus deformity

4) A 50-year-old woman with rheumatoid arthritis and medial joint space narrowing

 

Results of unicompartmental arthroplasty in the treatment of knee arthritis are highly dependent upon proper patient selection. The ideal candidate for unicompartmental arthroplasty is over the age of 60, of average weight, low demand, has good range of motion, and does not have a significant fixed valgus or varus deformity.Correct Answer: A 70-year-old, 70 kg man with medial knee arthritis and a 130° arc of motion

 

 

4057. (224) Q14-333:

Which of the following describes the desired alignment of the lower extremity following total knee arthroplasty:

 

1) Mechanical axis of 7° of valgus

3) Anatomic axis of 7° of valgus

2) Mechanical axis of 7° of varus

5) Anatomic axis of 7° of varus

4) Neutral anatomic axis

 

The anatomic axis is the angle of intersection of the shaft of the femur with the shaft of the tibia. The mechanical axis is the angle of intersection of a line connecting the center of the femoral head to the center of the tibial plateau, and a line connecting the center of the tibial plateau to the center of the ankle. Desired alignment following total knee arthroplasty is neutral mechanical axis or an anatomic axis of 5° to 7° of valgus.Correct Answer: Anatomic axis of 7° of valgus

 

 

 

4058. (225) Q14-334:

After appropriate balancing of soft tissues during total knee arthroplasty, the knee is loose in flexion and tight in extension. Appropriate management at this time would be:

 

1) Resection of more of the posterior femoral condyles

3) Resection of more of the proximal tibia

2) Resection of more of the distal femur

5) Insertion of a thinner polyethylene

4) Hamstring and gastroc-soleus lengthening

 

Balancing of the flexion/extension gaps in total knee arthroplasty should first be done by correcting the soft tissues. If the knee remains tight in flexion and extension after insertion of an 8-mm polyethylene spacer, more proximal tibia should be resected to increase both the flexion and extension gaps. If the knee is tight only in extension, more distal femur should be resected to increase only the extension gap. If the knee is tight only in flexion, a larger posterior slope can be cut into the tibia.Correct Answer: Resection of more of the distal femur

 

 

 

4059. (228) Q14-337:

Following total knee arthroplasty, how much knee flexion is required to arise from a chair?

 

1) 60°

3) 90°

2) 75°

5) 120°

4) 105°

 

At least 90° of knee flexion is required to easily arise form a chair.Correct Answer: 90°

 

 

 

4060. (4038) Q14-338:

A posterior cruciate substituting total knee prosthesis should be strongly considered in all of the following patients except:

 

1) A patient with a prior posterior cruciate ligament tear

3) A patient with rheumatoid arthritis

2) A patient with a 15° fixed varus deformity

5) A patient with a 10° fixed valgus deformity

4) A patient with a prior patellectomy

 

History of rheumatoid arthritis, prior patellectomy, fixed valgus deformity greater than or equal to 15°, or prior posterior knee instability are relative indications for use of a posterior cruciate substituting prosthesis in total knee arthroplasty. Outside of these settings, neither posterior cruciate preserving nor substituting designs has any definitive advantage over each other.Correct Answer: A patient with a 10° fixed valgus deformity

 

 

4061. (229) Q14-339:

All of the following are factors associated with increased polyethylene wear following total knee arthroplasty except:

 

1) Gamma sterilization in air

3) More conforming tibial articulating surface

2) Thin polyethylene thickness

5) Male patient gender

4) Younger patient age

 

More conformity between the femoral and tibial components results in less polyethylene wear, although at a cost of a lesser arc of motion. Ethylene oxide sterilization is preferred to gamma sterilization in air because of effects on wear characteristics. Younger patient age, male gender, and increased patient weight have all been shown to increase polyethylene wear following total knee arthroplasty.Correct Answer: More conforming tibial articulating surface

 

 

 

4062. (525) Q16-727:

When performing shoulder arthroplasty for rheumatoid arthritis, the following type of component should be employed:

 

1) Standard length, cemented humeral stem without a cement restrictor

3) Standard length, press-fit humeral stem

2) Standard length, cemented humeral stem with a cement restrictor

5) Long, cemented humeral stem without a cement restrictor

4) Long, press-fit humeral stem

 

When performing proximal humeral arthroplasty in patients with rheumatoid arthritis, the bone is frequently osteoporotic. Press-fit humeral stems have been associated with a high incidence of loosening. Unless an intraoperative fracture is encountered, long-stem prostheses are not necessary. A cement restrictor is used in this patient population because of the possible need of elbow arthroplasty in the future.Correct Answer: Standard length, cemented humeral stem with a cement restrictor

 

 

4063. (526) Q16-728:

An absolute contraindication to glenoid resurfacing when performing shoulder arthroplasty is:

 

1) Patient is less than or equal to 50 years of age

3) Insufficient bone stock

2) Presence of a small supraspinatus tear

5) Presence of an inflammatory arthropathy

4) Presence of osteonecrosis of the humeral head

 

Sufficient bone stock must be present to implant a glenoid component when performing shoulder arthroplasty. While hemiarthroplasty in a young patient without arthritic changes of the glenoid can be considered, age is not considered an absolute contraindication to glenoid resurfacing. While the presence of a large rotator cuff tear represents a contraindication to glenoid resurfacing because of the ârocking horseâ effect, which results in glenoid loosening, a small reparable rotator cuff tear does not prohibit resurfacing. Glenoid resurfacing is not contraindicated in osteonecrosis or rheumatoid arthritis provided there is a competent rotator cuff.Correct Answer: Insufficient bone stock

 

 

4064. (527) Q16-729:

Function following shoulder arthroplasty for rheumatoid arthritis is most dependent upon:

 

1) The preoperative status of the rotator cuff

3) Surgeon experience

2) Glenoid resurfacing

5) Use of a large head monopolar humeral component

4) Use of a bipolar humeral component

 

While pain relief following shoulder arthroplasty for rheumatoid arthritis is favorable, postoperative function is less predictable. The most important factor in predicting postoperative function is preoperative function, specifically as it relates to the status of the rotator cuff musculature.Correct Answer: The preoperative status of the rotator cuff

 

4065. (528) Q16-730:

Pain relief is more reliable in the treatment of rheumatoid arthritis of the shoulder using:

 

1) Synovectomy

3) Resection arthroplasty

2) Hemiarthroplasty

5) Total shoulder arthroplasty

4) Bipolar arthroplasty

 

Total shoulder arthroplasty most reliably relieves pain in patients with advanced rheumatoid arthritis of the shoulder. However, glenoid resurfacing should only be used in these patients in the presence of a competent rotator cuff and sufficient glenoid bone stock.Correct Answer: Total shoulder arthroplasty

 

 

4066. (529) Q16-731:

All of the following are risk factors for development of osteonecrosis of the humeral head except:

 

1) Use of fluoroquinolone antibiotics

3) Ethanol abuse

2) Sickle cell anemia

5) Paraplegia

4) Gaucher disease

 

Although fluoroquinolone antibiotics have been implicated as a risk factor for tendon ruptures, they have not been linked to the development of osteonecrosis. Paraplegia has been linked to the development of osteonecrosis of the humeral head, presumably from increased load in the weight-bearing dome of the humeral head.Correct Answer: Use of fluoroquinolone antibiotics

 

 

4067. (530) Q16-732:

After prosthetic replacement of the humeral head following a comminuted proximal humerus fracture, a poor outcome is most commonly associated with:

 

1) Not resurfacing the glenoid

3) Migration of the lesser tuberosity

2) Migration of the greater tuberosity

5) Presence of pre-existing glenoid arthritis

4) Excessive prosthetic anteversion

 

The factor most commonly associated with a poor result following prosthetic replacement of the humeral head following a fracture is migration of the greater tuberosity. The glenoid is generally not resurfaced. Excessive prosthetic retroversion is more closely associated with a poor result than excessive anteversion.Correct Answer: Migration of the greater tuberosity

 

 

4068. (531) Q16-733:

"Tuberosity escape" following humeral arthroplasty for a comminuted fracture of the proximal humerus is most commonly related to:

 

1) A humeral component that is too proud and retroverted

3) A humeral component that is too proud and anteverted

2) A humeral component that is too low and anteverted

5) A humeral component that is too low and has appropriate version

4) A humeral component that is too low and retroverted

 

Migration of the greater tuberosity is most commonly related to a prosthesis that has been placed too proud and in excessive retroversion.Correct Answer: A humeral component that is too proud and retroverted

 

4069. (532) Q16-734:

Positioning of the humeral stem at the time of total shoulder arthroplasty should allow congruent articulation with the glenoid component. Congruent articulation occurs in most shoulders with a humeral stem positioned in:

 

1) Neutral version

3) 20° to 30° of retroversion

2) 10° to 20° of retroversion

5) 20° to 30° of anteversion

4) 10° to 20° of anteversion

 

It is important to place the humeral stem in appropriate version to "mate" with the glenoid component. This is most often represented by 20° to 30° of humeral retroversion.Correct Answer: 20° to 30° of retroversion